Download as pdf or txt
Download as pdf or txt
You are on page 1of 132

CASE STUDIES IN CLINICAL BIOCHEMISTRY

Clinical Biochemistry is about patients – how we investigate their


signs and symptoms, how we diagnose their illnesses and how
we treat them. In this book the authors present a series of
clinical cases, all based on real patients, and invite the reader to
answer key questions using their knowledge and experience of
each topic. Each case and its questions are accompanied by the
authors’ detailed answers, which can be found by simply turning
the page. As such, it is an ideal revision aid for those studying
Medicine, Nursing and Biomedical Sciences and for those
preparing for post-graduate membership examinations.

The AUTHORS

Michael J. Murphy FRCP Edin, FRCPath


is Clinical Senior Lecturer in Biochemical Medicine at the University of Dundee. He
graduated in biochemistry and medicine from Trinity College Dublin, and subsequently
completed higher specialist training in Glasgow. He is Associate Editor of the Annals of
Clinical Biochemistry, a leading specialty journal. You can find out more about him and
his work at www.dundee.ac.uk/medschool/staff/michael-murphy/

Rajeev Srivastava MBBS, MS, FRCS, FRCPath


is Consultant Clinical Biochemist at the Royal Hospital for Sick Children and Southern
General Hospital Glasgow. He graduated from Delhi University and completed his
higher specialist training in Dundee. He has authored various publications in leading
medical journals and is co-author of the chapter on metabolic diseases in Hutchison’s
Paediatrics. His special interests lie in mineral metabolism, diagnosis of inborn errors of
metabolism and parenteral nutrition.

Allan Gaw, MD, PhD, FRCPath, FFPM, PGCert Med Ed


is Professor and Director of the Clinical Research Facility at Queen’s University,
Belfast. He is the author or editor of 18 books mostly on the subjects of Clinical
Biochemistry, Cardiovascular Research, Lipid Metabolism and Clinical Trials. With
Michael J. Murphy he co-wrote the best-selling textbook Clinical Biochemistry: An
Illustrated Colour Text, published by Churchill Livingstone (Elsevier), which has sold more
than 30,000 copies and has been translated into French, Spanish, Italian, Portuguese,
Chinese, Japanese and Greek. You can find out more about him and his work at
www.allangaw.com

i
ii
CASE STUDIES IN
CLINICAL BIOCHEMISTRY

Michael J. Murphy
Rajeev Srivastava
Allan Gaw

iii
First published 2012
By SA Press
SAPress42@gmail.com

Printed and bound in the United Kingdom by Clydeside Press

© 2012 Michael J. Murphy, Rajeev Srivastava and Allan Gaw

All rights reserved. No part of this book may be reprinted or reproduced or utilised
in any form or by any electronic, mechanical, or other means, now known or hereafter
invented, including photocopying or recording, or in any information retrieval system,
without permission in writing from the publishers.

British Library Cataloguing in Publication Data


A catalogue record for this book is available from the British Library

ISBN 978-0-956-3242-4-5

The publisher and the authors have no responsibility for the persistence or accuracy of
URLs for external or third party internet sites referred to in this book, and do not
guarantee that any content on such websites is, or will remain, accurate or
appropriate.

Note
Knowledge in all the areas of medicine described in this book is constantly changing
and improving. As new research and experience broaden our knowledge, changes in
practice, investigation, treatment and drug therapy may become necessary or
appropriate. Readers are advised to check the most current information provided (i)
on procedures featured or (ii) by the manufacturer of each product to be
administered, to verify the recommended dose or formula, the method of
administration and contraindications. It is the responsibility of the practitioner, relying
on their own experience and knowledge of the patient, to make diagnoses, to
determine dosages and the best treatment for each individual patient, and to take all
appropriate safety precautions. To the fullest extent of the law, neither the publisher
nor the authors assume any liability for any injury and/or damage to persons or
property arising out of, or related to, any use of the material contained in this book.
The Publisher

iv
Dedication
For those who taught us –
our teachers, our students and our patients.

v
CONTENTS

DEDICATION ................................................................. V
PREFACE ..................................................................... VII
ACKNOWLEDGEMENTS ................................................ IX
HOW TO USE THIS BOOK ............................................ X
GLOSSARY.................................................................... XI
FLUID AND ELECTROLYTE DISORDERS ......................... 1
ACID-BASE .................................................................... 9
RENAL ......................................................................... 19
CALCIUM AND BONE DISEASE .................................... 27
LIVER FUNCTION ........................................................ 35
THYROID FUNCTION .................................................. 43
ADRENAL FUNCTION .................................................. 51
DIABETES .................................................................... 59
PITUITARY FUNCTION ................................................ 69
LIPIDS ......................................................................... 73
TOXICOLOGY.............................................................. 79
PAEDIATRICS .............................................................. 83
MISCELLANEOUS ......................................................... 89
APPENDIX- ACID BASE STRATEGY ........................... 109
SUGGESTIONS FOR FURTHER READING .................... 110
USEFUL WEBSITES.................................................... 116

vi
PREFACE

This book is for those studying Clinical Biochemistry as part of their


medical, nursing or biomedical science courses. It will also be an excellent
revision aid for those preparing for post-graduate and professional
examinations such as the MRCP. What it is not is a textbook of Clinical
Biochemistry – we have already written that and it is now in its 4th edition.
Although that book – Clinical Biochemistry: An Illustrated Colour Text –
contained clinical case studies many of our readers and our students have
asked for more to use as additional learning aids and for revision in
preparation for examinations. In response, we have prepared a
comprehensive set of clinical cases with answers and explanations that
cover a range of topics in our subject.
We have assumed throughout that you have already studied each of the
topics and are ready to put your knowledge into practice. Clinical
Biochemistry is about physiology and pathology and, above all, the
practice of medicine. It is a vital component of modern diagnostics and
patient management. Unless we have a thorough understanding of the use
of biochemical tests in clinical practice we will ultimately fail to do the best
for our patients.
The clinical cases in this book are all based on real life patients and each
presents a challenge, just as every patient we encounter in practice may be
thought of as a puzzle to be solved. By looking at biochemical results in
the context of a clinical history, examination findings and other diagnostic
information we can begin to unravel the puzzle before us. If we have
enough information we can propose a differential diagnosis, a treatment
plan and a prognosis.
For each case we provide a solution to the problem and we give
explanations for each of our answers to aid your learning and revision.

We have not dwelt on laboratory practices preferring to place our subject


firmly in the clinical arena where it belongs. Of course, what happens in
the laboratory – how we perform the tests and how we ensure their quality
– is of paramount importance, but it need not be the concern of most
clinicians requesting biochemical services. They should be able to take the
vii
excellence of the laboratory service for granted, and be able to focus on
the clinical problems for which they are seeking help.

As such, in this book, you will not find questions about how to measure
blood glucose or plasma cholesterol or urinary creatinine, but you will be
asked to interpret the results of such analyses in their clinical context.

Finally, if you wish to explore further any of the topics dealt with in this
book, we have also included a list of useful additional resources (books
and websites) that provide detailed specialist coverage.

Michael J. Murphy, Rajeev Srivastava & Allan Gaw


Dundee, Glasgow & Belfast, 2012

viii
ACKNOWLEDGEMENTS

The idea for this book of case studies in Clinical Biochemistry came out
of discussions with colleagues and students over a number of years. In
particular, we must acknowledge the debts we owe to:

 Michael Burns and Laura Hughes, who kindly read and


commented on early drafts of the case studies and answers in this
book.

 Peter Galloway and Colin Fletcher for their help and advice in
sourcing appropriate case material.

 Liz Ronald for her help in taking this manuscript from its first
drafts to the finished book.

 David Tolmie for his help with the cover design.

 Anne Hicks for preparing the Kindle version of the book.

Despite their valuable input we emphasise, however, that any factual


errors or pieces of clumsy prose that remain are entirely our own.

Finally, we would like to thank our editor at SA Press, Moira Mungall,


who with the use of carrots and sticks in equal measure encouraged us to
finish this book.

ix
HOW TO USE THIS BOOK

This book is designed to be very simple to use. On each odd-numbered


page you will find a clinical case study. This will usually consist of a short
clinical history, some clinical findings, some biochemical results and some
questions. For each case you are asked to consider your answers and then
you can compare yours with ours. You do not have to rummage in the
back of the book matching case numbers to answers – finding the
solutions is as simple as turning the page, for you will find our solutions
and their explanations overleaf on each even-numbered page.

To interpret any biochemical results you will need to have access to


appropriate reference ranges. For the results in this book we have
provided these in brackets after each result. While this is repetitious, it
avoids you having to search for a list of reference ranges every time you
need one. Please remember, however, that reference ranges vary from
laboratory to laboratory. When evaluating biochemical results you should
always use the ranges published by the institution performing the analyses.
If in doubt contact your local laboratory for further information.

We have also used the convention throughout of a capital ‘L’ to denote


litre. This is simply to avoid any confusion with a lower case ‘ l ’ and the
number ‘ 1 ’. As you can see they are very similar. Unless otherwise stated
all results are from analyses of venous blood.

x
GLOSSARY
A&E Accident and Emergency
ACTH Adrenocorticotrophic Hormone
ADH Anti-diuretic Hormone
Alk Phos Alkaline Phosphatase
ALT Alanine Aminotransferase
AST Aspartate Aminotransferase
AVP Arginine Vasopressin
BMD Bone Mineral Density
BMI Body Mass Index
BNF British National Formulary
BP Blood Pressure
CA-125 Carbohydrate Antigen-125
CAH Congenital Adrenal Hyperplasia
CBD Common Bile Duct
CKD Chronic Kidney Disease
COPD Chronic Obstructive Pulmonary Disease
CNS Central Nervous System
CRP C-reactive Protein
DDAVP Desmopressin – Synthetic Arginine Vasopressin
DHA Dehydroepiandrosterone
eGFR Estimated Glomerular Filtration Rate
E2 Oestradiol
ESR Erythrocyte Sedimentation Rate
FAO Fatty Acid Oxidation

xi
GLOSSARY
f T4 Free Thyroxine
FSH Follicle Stimulating Hormone
GGT Gamma-glutamyl Transpeptidase
GnRH Gonadotrophin Releasing Hormone
GP General Practitioner
[H+] Hydrogen-ion Concentration
Hb Haemoglobin
HGH Human Growth Hormone
IEM Inborn Errors of Metabolism
IFG Impaired Fasting Glycaemia
IGF-1 Insulin-like Growth Factor-1
IGT Impaired Glucose Tolerance
IM Intramuscular
ITU Intensive Therapy Unit
IV Intravenous
K+ Potassium-ion
LFT Liver Function Tests
LH Luteinizing Hormone
MIBG Meta-iodobenzylguanidine
MRI Magnetic Resonance Imaging
mmHg Millimetres of Mercury
MSH Melanocyte Stimulating Hormone
Na+ Sodium-ion
pH Negative Log of Hydrogen-ion Concentration

xii
GLOSSARY

PSA Prostatic-specific Antigen


PTH Parathyroid Hormone
PTHRP PTH-related Peptide
PCO2 Partial Pressure of Carbon Dioxide
PO2 Partial Pressure of Oxygen
SD Standard Deviation
SHBG Sex Hormone Binding Globulin
SIAD Syndrome of Inappropriate Antidiuresis
T4 Thyroxine
TFT Thyroid Function Tests
TRH Thyrotrophin Releasing Hormone
TSH Thyroid Stimulating Hormone
U&E Urea and Electrolytes
WCC White Cell Count

xiii
xiv
FLUID AND ELECTROLYTE DISORDERS CASE 1

An 82-year-old woman is discovered by her family, lying on the floor at


home. She is found to have a left hemiparesis and a stroke is diagnosed.
Clinical assessment of her volume status indicates dehydration: pulse 112
beats per minute, blood pressure 95/65 mmHg, lax skin and dry tongue.
The following biochemical results were obtained on admission:

Na+ 169 mmol/L (135-145)


K+ 3.9 mmol/L (3.4-4.9)
Urea 23.5 mmol/L (2.5-8.0)
Creatinine 160 µmol/L (40-130)
Glucose 7.6 mmol/L (4.0-5.5)

(a) Describe and comment on the biochemical abnormalities.


(b) Give an assessment of the patient’s water status and sodium status.
(c) In the light of your answer to the previous question, predict the urine
osmolality and urinary sodium concentration.
(d) How would you treat her fluid/electrolyte status?

1
FLUID AND ELECTROLYTE DISORDERS ANSWER 1

(a) Severe hypernatraemia; urea and creatinine also raised, urea more
than creatinine. These biochemical abnormalities are frequently seen
in clinical scenarios where fluid intake is reduced (as here), or, less
frequently, when fluid loss is increased. Glucose is raised but may
just reflect insulin resistance acquired from the stress response,
which is associated with increased concentrations of anti-insulin
hormones like cortisol and adrenaline.

(b) Water status is reduced; the evidence for this is primarily clinical
(tachycardia, hypotension), reflecting reduced extracellular fluid
(ECF) volume, although the greater rise in urea (compared with
creatinine) is also consistent with dehydration. Patients are often less
clinically dry than expected given the severity of the hypernatraemia.
This is because water loss is distributed across all body
compartments. Na+ intake will have been reduced for the same
reason as water intake. However, some of the homeostatic
mechanisms designed to protect blood volume (secondary
hyperaldosteronism) minimise Na+ loss, even as insensible losses of
pure water from respiration continue. Hence the relative loss of
more water than Na+, as evidenced by high serum Na+
concentration.

(c) Urine osmolality will be high (>600 mmol/kg), reflecting urine


concentration due to ADH secretion, stimulated by hypovolaemia
(non-osmotic stimulus) and hypernatraemia (osmotic stimulus).
Urinary Na+ concentration will be very low (<10 mmol/L)
reflecting sodium retention due to secondary hyperaldosteronism,
precipitated again by hypovolaemia.
(d) She needs water and was given 5% dextrose IV (she could not drink
initially). This is isotonic with plasma; pure water would precipitate
intravascular haemolysis because of the osmotic difference across
red cell membranes. Isotonic (0.9%) saline replaces both Na+ and
water and does not correct a primary water deficiency.

2
FLUID AND ELECTROLYTE DISORDERS CASE 2

A 64-year-old male smoker presents with unintentional weight loss of 20


kg over three months. On review of systems he admits to intermittent dry
cough. On chest examination he has signs consistent with left lower lobe
consolidation; water (volume) status is unremarkable. Chest X-ray
confirms consolidation and also reveals hilar lymphadenopathy. Lung
cancer was provisionally diagnosed. Before bronchoscopy, urea and
electrolytes were measured:

Na+ 124 mmol/L (135-145)


K+ 3.5 mmol/L (3.4-4.9)
Urea 5.8 mmol/L (2.5-8.0)
Creatinine 110 µmol/L (40-130)

(a) Describe and explain the biochemical abnormalities.

(b) Give your assessment of his water and sodium status.

(c) How would you treat his fluid and electrolyte abnormalities?

3
FLUID AND ELECTROLYTE DISORDERS ANSWER 2

(a) The patient has hyponatraemia; potassium is low-normal.


Importantly serum urea and creatinine are within the respective
reference ranges, suggesting that glomerular function is normal.

(b) Water status is probably increased. Why? The most important


information here is his volume status, reportedly ‘unremarkable’ –
effectively telling you that he is neither dry nor oedematous – in
turn excluding sodium depletion and effective circulating volume
depletion as the basis for his hyponatraemia. In the context of the
information provided, the syndrome of inappropriate antidiuresis
(SIAD) should be suspected. ADH secretion stimulated by non-
osmotic stimuli causes water to be retained, causing a ‘dilutional’
hyponatraemia. The cause of SIAD here is likely to be lung
cancer, although other chest pathologies may also cause it.
Volume status may be unremarkable because the water excess is
distributed evenly across all body compartments. He probably has
relatively normal sodium status – there is no information
suggesting a reason for this to be altered.

(c) Symptoms of hyponatraemia (usually neurological) do not usually


develop if sodium is much above 120 mmol/L. If intravenous
fluids are required to correct severe hyponatraemia, isotonic
(0.9%) saline is preferred; this helps to restore the balance of
sodium and water (reduces relative sodium deficit). Less severe
hyponatraemia due to SIAD is treated with fluid (water)
restriction (usually to less than a litre a day). This effectively
requires hospital admission since it is virtually impossible to
supervise elsewhere. In this case it would be reasonable to
monitor sodium and admit for treatment only if sodium fell to
<120 mmol/L and/or the patient developed symptoms of
hyponatraemia.

4
FLUID AND ELECTROLYTE DISORDERS CASE 3

A 25-year-old male cyclist is admitted to a neurosurgical unit after


sustaining severe head injuries in a road traffic accident. He develops
polyuria (urine output in excess of 15 L) over the next 24 h. Serum sodium
climbs from 148 mmol/L on admission, to 168 mmol/L on day 4, at
which point a spot urine osmolality is 80 mmol/kg.

(a) What is the diagnosis?

(b) How would you treat his hypernatraemia?

5
FLUID AND ELECTROLYTE DISORDERS ANSWER 3

(a) Cranial diabetes insipidus. This patient has sustained traumatic damage
to his posterior pituitary gland (which produces ADH). As a result he
is unable to retain water. The diagnosis can effectively be made on the
basis of the history. The urine osmolality simply confirms his inability
to concentrate urine (urine osmolality should be well in excess of 600
mmol/kg in the face of severe hypernatraemia as here).

(b) He is losing large amounts of (almost pure) water and requires water
to be replaced. As he is unconscious this cannot be given orally and
must be given intravenously as 5% dextrose. The rate at which this
should be given will be determined by his urine output. He will also
require exogenous ADH since he is unable to produce this himself.
This is given as the vasopressin analogue DDAVP (AVP stands for
arginine vasopressin, another name for ADH). This can be
administered in a number of ways depending on the clinical context:
as nasal drops, orally or by injection (IV or IM).

6
FLUID AND ELECTROLYTE DISORDERS CASE 4

A 77-year-old woman with a history of stroke attends the hypertension


clinic; she is on four antihypertensive drugs. Ambulatory blood pressure
monitoring confirms refractory hypertension with mean daytime readings
172/98 mmHg. Furosemide 40 mg daily is added, and magnetic resonance
angiography requested to exclude renal artery stenosis. Three days after
furosemide is added, an out-of-hours doctor is called to see her. She is
clearly unwell and unable to give a clear history. Her daughter reports
increasing forgetfulness over the previous few months. On examination
she is clearly dehydrated with lax skin and dry mucosa. Serum urea and
electrolytes are as follows:

Na+ 127 mmol/L (135-145)


K+ 3.2 mmol/L (3.4-4.9)
Urea 15.0 mmol/L (2.5-8.0)
Creatinine 140 µmol/L (40-130)

(a) Describe and comment on the biochemical abnormalities.

(b) Give your assessment of the patient’s sodium and water status.

(c) How would you treat this patient?

7
FLUID AND ELECTROLYTE DISORDERS ANSWER 4

(a) The patient has hyponatraemia and hypokalaemia and evidence of


reduced glomerular function in the form of raised urea and creatinine.
The most likely explanation is that she has inadvertently overdosed on
her furosemide, resulting in excessive natriuresis and resultant diuresis.

(b) Total body water and sodium are likely to be decreased. This case
illustrates a golden rule of sodium and water balance: water follows
sodium everywhere. It also illustrates the diagnostic value of clinical
dehydration in the context of hyponatraemia. Where there is clear
evidence of decreased water status from clinical dehydration, the
finding of a reduced ratio of sodium to water is diagnostic of sodium
depletion – it allows for no other interpretation. Furosemide inhibits
reabsorption of sodium and chloride in the ascending loop of Henlé
(hence the term: loop diuretic) and distal renal tubule; it also blocks a
chloride-binding co-transport system. The effect is increased excretion
of water, sodium, chloride, magnesium, and calcium.

(c) She requires rehydration. Since she is sodium depleted, fluid


resuscitation should take the form of intravenous isotonic (0.9%)
saline initially.

8
ACID-BASE CASE 5

A 78-year-old man is admitted to hospital as an emergency because of


haematemesis. On arrival he is clinically shocked with pulse rate 112 beats
per minute and blood pressure 90/65 mmHg. His admission blood results
are:

Urea and Electrolytes

Na+ 143 mmol/L (135-145)


K+ 3.6 mmol/L (3.4-4.9)
Cl- 96 mmol/L (95-105)
HCO3- 5 mmol/L (21-28)
Urea 13.8 mmol/L (2.5-8.0)
Creatinine 130 µmol/L (40-130)

Arterial Blood Gases

H+ 97 nmol/L (35-45)
PCO2 2.7 kPa (4.5-5.6)
PO2 16.1 kPa (12-15)

(a) Classify the acid-base disorder.

(b) Explain the results.

(c) Is there anything else you would wish to measure?

9
ACID-BASE ANSWER 5

(a) [H+] is grossly elevated indicating a severe acidosis. Bicarbonate and


PCO2 are low, the bicarbonate much more profoundly. A low
bicarbonate is defined as a metabolic acidosis; a low PCO2 defined as a
respiratory alkalosis. The metabolic acidosis must therefore be
primary, and the respiratory alkalosis compensatory. So: partially
compensated metabolic acidosis.

(b) The history provides a clear indication of the cause of the metabolic
acidosis. The patient is reported to be clinically shocked, indicating
lack of perfusion of his tissues (invariably seen in severe blood loss, as
with haematemesis). This means that the tissues have to metabolise
fuel in the absence of oxygen, resulting in production of lactate.
Lactate is acidic, dissociating H+ and thereby releasing an acid load
into the circulation. The law of mass action results in the following
reaction being driven to the right: H+ + HCO3- ⇔H2CO3 ⇔H2O+
CO2. This results in the production of PCO2, which has to be blown
off (hence acidotic or Kussmaul breathing).
(c) Only two things cause severe metabolic acidosis with any frequency:
ketoacidosis and lactic acidosis. So you need to measure ketones and
lactate. Ketoacidosis is mostly associated with diabetes mellitus,
especially type 1, but can also be seen in alcoholic patients (alcoholic
ketoacidosis) and in biochemical starvation. In practice you would
dipstick his urine for ketones, rather than quantifying them in blood
(although increasingly clinicians do exactly that in managing diabetic
ketoacidosis). Do not forget that someone as ill as this patient may
have slightly raised glucose due to the anti-insulin hormones
associated with severe physiological stress (cortisol, adrenaline, growth
hormone), and indeed might also be mildly ketotic if he had not eaten
for a while. If diabetic ketoacidosis were to explain these results, you
would expect the glucose to be very high and the urine to be strongly
positive for ketones (at least three or four pluses). Practical point:
lactate is collected into the same specimen type as glucose. This
patient’s lactate was 19.0 mmol/L (reference range <2 mmol/L).

10
ACID-BASE CASE 6

A 63-year-old man is brought to Accident and Emergency after he goes


into cardiorespiratory arrest while eating out in a restaurant. On arrival
resuscitation measures are continued and the patient is briefly stabilised;
venous and arterial blood samples are collected. The results of the arterial
blood gases are shown below.

H+ 132 nmol/L (35-45)


PCO2 14.6 kPa (4.5-5.6)
HCO3- 8.5 mmol/L (21-28)
PO2 9.2 kPa (12-15)

(a) Describe the acid base abnormalities.

(b) Discuss the pros and cons of administering bicarbonate to treat a


profound acidosis as here.

11
ACID-BASE ANSWER 6

(a) This set of arterial blood gases is typical of a patient in extremis. [H+] is
grossly elevated (i.e. profound acidosis). The clinical context is of
cardiorespiratory arrest, so logically it does not matter which of PCO2
or bicarbonate is examined next. As anticipated in respiratory arrest,
the PCO2 is grossly elevated, reflecting severe respiratory acidosis. The
bicarbonate is very low, indicating a severe metabolic acidosis. The
fact that both components of the ratio in the Henderson-Hasselbalch
equation have changed in opposite directions indicates that we have
two primary processes occurring simultaneously. This is therefore a
mixed acid-base disturbance. This picture is a (profound) mixed
respiratory and metabolic acidosis.

(b) An acidosis of this severity is life-threatening, irrespective of its


underlying cause. It would be logical in this clinical context to
administer IV bicarbonate in an attempt to normalise the [H+].
Indeed, this used to be part of the management of acute cardiac arrest.
However, its use has been discontinued on the grounds that there is
no evidence of clinical benefit; indeed patients treated in such a way
may do worse. This has been postulated to be due to either
‘overshoot’ alkalosis (unlikely in this case with severe acidosis) or
hypercapnia (due to the increased production of CO2 from [H+] and
[HCO3-]), which requires adequate ventilation for elimination.

12
ACID-BASE CASE 7

A 57-year-old man is admitted because of breathlessness and wheezing.


He is known to suffer from chronic obstructive pulmonary disease;
medications included inhalers and oral prednisolone. He is also a heavy
smoker. On examination, he is breathless, cyanosed and pyrexial. His chest
is hyper-resonant and he has generally decreased air entry with bilateral
wheeze and coarse crepitations on the right side. Chest X-ray shows
probable collapse of the right upper lobe and emphysematous changes.
Arterial blood gases on air show:

H+ 56 nmol/L (35-45)
PCO2 11.3 kPa (4.5-5.6)
HCO3- 36 mmol/L (21-28)
PO2 7.1 kPa (12-15)

(a) Describe the acid base abnormality.

(b) What background acid-base picture might you anticipate if he was


not acutely breathless?

(c) Make up a set of acid-base results that would be consistent with your
answer to part (b).

13
ACID-BASE ANSWER 7

(a) [H+] is raised, indicating an acidosis. PCO2 is raised indicating


respiratory acidosis. Bicarbonate is elevated indicating metabolic
alkalosis. Thus, the respiratory acidosis must be primary and the
metabolic alkalosis compensatory. So, partially compensated
respiratory acidosis.

(b) It is worth delving more deeply into his acid-base status. His history
of COPD means that you would expect him to have a background
respiratory acidosis (raised PCO2), and if he was stabilised on
treatment, you would expect this to be completely (or almost
completely) compensated (raised bicarbonate). So, a compensated
respiratory acidosis is the most likely background picture.

(c) H+ 44 nmol/L (35-45)


PCO2 8.0 kPa (4.5-5.6)
HCO3- 33 mmol/L (21-28)
PO2 7.1 kPa (12-15)

These figures represent a compensated respiratory acidosis. H+,


PCO2 and bicarbonate are related mathematically in the Henderson-
Hasselbalch equation, and if the H+ and PCO2 are measured (which
they are) then the bicarbonate concentration can be deduced. This
is exactly what blood gas analysers do; hence the term ‘standardised
bicarbonate’ – it indicates that the machine has calculated, rather
than measured, the bicarbonate.

14
ACID-BASE CASE 8

An 80-year-old woman is admitted with a myocardial infarction. She is


not shocked or hypotensive. She is on acetazolamide for glaucoma.

Urea and Electrolytes


Na+ 139 mmol/L (135-145)
K+ 4.5 mmol/L (3.4-4.9)
Cl- 116 mmol/L (95-105)
HCO3- 12 mmol/L (21-28)
Urea 10 mmol/L (2.5-8.0)
Creatinine 120 µmol/L (40-130)

Arterial Blood Gases


H+ 59 nmol/L (35-45)
PCO2 3.9 kPa (4.5-5.6)
PO2 10.0 kPa (12-15)

(a) Define the acid-base abnormality.

(b) Why have chloride and bicarbonate been measured?

(c) Can you suggest a diagnosis?

15
ACID-BASE ANSWER 8

(a) [H+] is elevated indicating an acidosis. Bicarbonate and PCO2 are


low, the bicarbonate more so. A low bicarbonate is a metabolic
acidosis, a low PCO2 is a respiratory alkalosis. So the metabolic
acidosis must be primary, and the respiratory alkalosis
compensatory. Partially compensated metabolic acidosis. This case
is similar to, but much less severe than, the example given in Case 5.

(b) Chloride and bicarbonate have been measured to allow the anion
gap to be calculated. This is a biochemical tool that can be helpful in
finding the cause of a metabolic acidosis. Essentially, it is the
difference between the two main cations in the blood (Na+ and K+)
and the two main anions (Cl- and bicarbonate): [Na+ + K+] - [Cl- +
HCO3-]. K+ is sometimes omitted because it’s present in small
concentrations. What does the anion gap tell you? If it is high
(which it usually is) then not very much. However, if it is normal (8-
16 mmol/L), the list of causes is much smaller, and it is therefore
much more useful. This list includes renal tubular acidosis and
chronic diarrhoea or intestinal fistula.

(c) The finding of a normal anion gap, and the knowledge that the
patient was on acetazolamide for her glaucoma, provides the
explanation. Acetazolamide is a carbonic dehydratase inhibitor; its
action means that bicarbonate is not reabsorbed and is hence lost
from the body in the urine, causing a metabolic acidosis. In other
words it causes a special kind of ‘acquired’ renal tubular acidosis.

16
ACID-BASE CASE 9

A 56-year-old woman is admitted seriously ill and confused. Admission


biochemical results are shown below:

Na+ 135 mmol/L (135-145)


K+ 2.6 mmol/L (3.4-4.9)
Cl- 59 mmol/L (95-105)
HCO3- 53 mmol/L (21-28)
Urea 6.8 mmol/L (2.5-8.0)

Arterial Blood Gases


H+ 32 nmol/L (35-45)
PCO2 9.3 kPa (4.5-5.6)
PO2 Not Available (12-15)

Cor pulmonale and systemic oedema were diagnosed. The patient was on
furosemide.

(a) Describe the acid-base disorder.

(b) What is going on?

17
ACID-BASE ANSWER 9

(a) [H+] is slightly low indicating a mild alkalosis. Bicarbonate is very


high indicating a marked metabolic alkalosis, and the high PCO2
indicates a respiratory acidosis. In principle there are three
possibilities: (1) Primary respiratory acidosis with metabolic
compensation; (2) Metabolic alkalosis with respiratory compensation;
(3) Mixed acid-base disturbance with primary respiratory acidosis and
primary metabolic alkalosis. It cannot be (1) because this would break
a golden rule of acid-base balance – that over-compensation cannot
occur. (2) is also unlikely. There is a limit to the ability of PCO2 to
rise in compensation for a metabolic alkalosis – the hypoxia resulting
from the associated hypoventilation would override and drive the
respiratory centre to ventilate. So, this is likely to be (3) – a mixed
acid-base picture with two components (respiratory acidosis and
metabolic alkalosis) with the metabolic alkalosis slightly
predominating.

(b) The primary respiratory acidosis derives from the lung disease and
the oedema from cardiac failure. The coexisting metabolic alkalosis
remains to be explained; why is the bicarbonate so high? Secondary
hyperaldosteronism is almost certainly part of the explanation. It is
due here either to the effective hypovolaemia of an oedematous state
(cardiac failure), or diuretic therapy. Loop diuretics, such as
furosemide, increase Na+ delivery to the distal tubule, which
accentuates urinary K+ loss.

18
RENAL CASE 10

Compare and contrast the renal function of the following two patients:

1. A frail, 81-year-old woman who weighs 52 kg with a serum creatinine


of 109 µmol/L and an eGFR that has been calculated as 44
mL/min/1.73m2.

2. A muscular, 24-year-old man who weighs 89 kg with a serum creatinine


of 104 µmol/L and an eGFR that has been calculated as >60
mL/min/1.73m2.

(a) What is the purpose of estimating GFR?

(b) Explain why these patients have similar serum creatinine


concentrations, but very different renal function.

19
RENAL ANSWER 10

(a) Serum creatinine varies inversely with glomerular function. However,


its ability to detect minor reductions in GFR is limited. This is
because it does not take into account a number of confounding
variables including age, sex and muscle mass. Several of these
variables are, however, taken into account in the formula that is
widely used to estimate GFR. This formula, therefore, allows for the
detection of milder degrees of renal dysfunction that cannot readily
be detected on the basis of serum creatinine alone.

(b) These two patients represent extremes of the relevant confounding


variables discussed above. Based on these variables the first patient
would be expected to have a substantially lower serum creatinine if
her renal (glomerular) function was normal. Serum creatinine in her
case is significantly higher and represents an abnormal level. By
contrast, a similar serum creatinine concentration in the second
patient represents normal renal function.

It is worth noting that many older patients have reduced eGFR. It is


not clear whether this simply reflects the normal ageing process as
opposed to widespread prevalence of specific renal pathologies, e.g.
atherosclerotic vascular disease.

20
RENAL CASE 11

A 38-year-old man was admitted with a crush injury following a road


traffic accident. On admission he was clinically shocked; a tentative
diagnosis of internal bleeding was made. At emergency exploratory
laparotomy his abdominal cavity was found to be full of blood. He was
admitted to the surgical high-dependency unit post-operatively, during
which his renal function was monitored.

Post-op Creatinine Serum Osmolality Urine Osmolality


µmol/L mmol/kg mmol/kg
Day 1: 270 302 484
Day 2: 330 305 372
Day 3: 350 304 310

(a) What is the purpose of requesting serum and urine osmolality


measurements?

(b) How would you interpret the osmolality results on (i) day 1 and (ii)
day 3?

(c) What would you expect his estimated glomerular filtration rate
(eGFR) to be (in general terms, not a specific figure)?

21
RENAL ANSWER 11

(a) Simultaneous measurement of serum and urine osmolality allows


evaluation of renal tubular function. The more different these are
from each other, the more convincing the evidence of tubular action
on the filtrate. Urine sodium concentration is sometimes requested
for a similar reason, the logic being that a low or undetectable urine
sodium indicates renal tubular reabsorption of sodium, usually as a
result of aldosterone action, in turn stimulated by hypovolaemia.
(b) The ratio of urine/serum osmolality on day 1 is 1.6. This indicates
some degree of tubular action. By contrast the ratio is close to 1 on
day 3, suggesting that the renal tubules are doing very little to the
filtrate in terms of water reabsorption. This in turn suggests that the
patient has developed intrinsic renal damage as a result of poor
perfusion due to blood loss. Renal replacement treatment may be
required if his kidney function does not recover.
(c) In general terms his eGFR will be reduced. Estimation of GFR adds
little here - there is clear evidence from his elevated serum creatinine
that GFR is low. Indeed, estimation of GFR is not advised in patients
whose renal function is changing rapidly – the accuracy of the
estimation is compromised. Laboratory computer systems are not
usually able to take this into account and will issue an eGFR
automatically whenever creatinine is reported on an adult patient, so
vigilance in interpretation is required from requesting clinicians.

22
RENAL CASE 12

An 18-year-old man who had suffered a flu-like illness for the previous
two weeks is seen by his GP. Clinical examination includes urinalysis
which indicates that his urine is strongly positive for protein on dipstick
testing. A 24-hour urine collection done to confirm the dipstick finding
shows gross proteinuria of 3g/24hours. The patient has pitting oedema of
both ankles and his blood pressure is 142/84 mmHg. He is referred
urgently for renal review and his baseline urea and electrolytes are as
follows:

Na+ 127 mmol/L (135-145)


K+ 4.8 mmol/L (3.4-4.9)
Urea 11.6 mmol/L (2.5-8.0)
Creatinine 152 µmol/L (40-130)

Renal biopsy confirmed an acute glomerulonephritis.

(a) Describe and comment on the biochemical abnormalities.

(b) Give your assessment of the patient’s sodium and water status.

(c) Why does the patient have oedema?

23
RENAL ANSWER 12

(a) The key biochemical finding is gross proteinuria, as evidenced by the


24-hour urine collection. The patient also has hyponatraemia as well as
evidence of reduced glomerular function in the form of raised urea
and creatinine. High-normal potassium is likewise consistent with
reduced glomerular function.

Proteinuria of this degree is diagnostic of nephrotic syndrome and will


be associated with low serum protein concentration; protein synthesis
is unable to compensate for the rate of protein loss. Serum total
protein was 47g/L and serum albumin 23 g/L. (Reference ranges are
50-70 and 40-52 respectively.)

(b) Total body water and sodium are both likely to be increased. As a
result of the low plasma protein, the balance of osmotic and
hydrostatic forces at the level of the capillaries is altered so that there
is net outflow of water. This results in excess fluid in the extracellular
compartment and reduction of the plasma volume (usually referred to
as effective circulating volume depletion). The latter precipitates both
increased ADH secretion (resulting in water retention) and secondary
hyperaldosteronism (resulting in sodium and water retention). Both of
these homeostatic responses restore the plasma volume towards
normal. However, the fluid retained by these mechanisms continues to
be distributed unevenly for the reasons outlined above and a vicious
circle is created. ADH induces retention of ‘pure’ water, i.e. not with
sodium – hence the hyponatraemia despite increased total body
sodium.

(c) The accumulation of excess fluid in the extracellular compartment


gives rise to the clinical signs of pitting oedema. Its presence may be
considered a short-hand for sodium and water excess and also has a
relatively short differential diagnosis. It is therefore an extremely
useful clinical sign.

24
RENAL CASE 13

A 57-year-old man is seen by his GP for management of his cardiovascular


risk. Modifiable risk factors identified include: the fact that he smokes
twenty cigarettes daily; his weight which corresponds to a body mass index
(BMI) of 33.9 kg/m2. He is also hypertensive; mean daytime readings on
ambulatory blood pressure monitoring are 152/89 mmHg. Cholesterol is
4.6 mmol/L. On systems review he admits to symptoms suggestive of
intermittent claudication. He is counselled about smoking, given dietary
advice, and ramipril 2.5 mg daily is prescribed.

(a) What class of drugs does ramipril belong to? How do they work?

(b) What advice is routinely given regarding the commencement of


ramipril?

(c) What pathology underpins this advice and how might you investigate
it?

25
RENAL ANSWER 13

(a) Ramipril is an angiotensin converting enzyme inhibitor (ACEI). ACE


is involved in the conversion of angiotensin I to angiotensin II, one
step of the renin-angiotensin-aldosterone pathway that begins with
the production of renin and ends with the action of aldosterone on
the renal tubules. The effects of blocking this pathway by ACE
inhibition are multiple but include reducing arterial resistance and
hence blood pressure.

(b) Renal function including serum potassium should be checked before


commencing patients on ACEI, and within a week or so of
commencing it; ACEI can precipitate a sharp reduction in glomerular
function. The mechanism of this is not entirely clear although the
following is plausible: angiotensin II vasoconstricts the efferent
arterioles in the glomerulus, thereby helping to maintain perfusion
and GFR. In renal artery stenosis (RAS), afferent flow is
compromised; the kidney becomes increasingly dependent on the
angiotensin II mechanism to maintain filtration. In this context, ACE
inhibition reduces perfusion thereby reducing filtration (GFR).

(c) The pathology of concern is renovascular hypertension, i.e. atheroma


affecting the renal arteries, especially at their openings off the
abdominal aorta. If further investigation is required, magnetic
resonance angiography is the optimal way of imaging this. Plasma
renin concentration will also be grossly elevated in RAS as reduced
glomerular perfusion greatly stimulates its production. RAS is often
associated with atheroma affecting other arterial trees (for example
lower limb and coronary), so the history of intermittent claudication
here is relevant. Heavy smoking is also a risk factor for Buerger’s
disease (thromboangiitis obliterans), which causes inflammation and
blockage of small and medium-sized arteries and often results in limb
amputations.

26
CALCIUM AND BONE DISEASE CASE 14

A 48-year-old woman is admitted with a humeral fracture following


minimal trauma. Biochemical investigations included a bone profile,
including plasma phosphate and magnesium, the results of which are
shown below. X-ray of her humerus reveals osteolytic lesions in addition
to the fracture.

Total Calcium 3.28 mmol/L (2.20-2.60)


Adj Calcium 3.38 mmol/L (2.20-2.60)
Phosphate 0.42 mmol/L (0.7-1.4)
Albumin 42 g/L (40-52)
Alk Phos 532 U/L (80-280)
Magnesium 0.73 mmol/L (0.7-1.0)

(a) Suggest a differential diagnosis for this clinical presentation.

(b) What further investigations would be appropriate?

27
CALCIUM AND BONE DISEASE ANSWER 14

(a) The key phrase in this clinical presentation is ‘minimal impact’.


Fractures are not uncommon, but low-impact fractures are rare and
should prompt further investigation to establish if they have
occurred in osteoporotic bones (where bone mineral density is
reduced) or bones affected by other disease states e.g. metastases
(so-called ‘pathological fractures’).
The differential diagnosis here includes malignancy of bone (either
primary or secondary), including multiple myeloma, and primary
hyperparathyroidism. The finding of severe hypercalcaemia is
consistent with any of these diagnoses, as are the osteolytic lesions.
(b) (i) Parathyroid hormone (PTH): should help distinguish between
hyperparathyroidism (PTH high) and malignant causes (PTH
suppressed). In the latter, hypercalcaemia is driven by PTHRP, a
peptide that has biological actions similar to PTH (hence the name
– ‘PTH-related peptide’) but which does not cross-react in the PTH
immunoassay.
(ii) 25-(OH)-vitamin D: Osteomalacia might explain raised alkaline
phosphatase and fracture, but is less easily reconciled with the
hypercalcaemia (low vitamin D would be expected to be associated
with hypocalcaemia). However, in hyperparathyroidism, coexistent
vitamin D deficiency may contribute to the severity of the skeletal
lesions.
(iii) Bone mineral densitometry: will help to establish if patient’s
bone mineral density (BMD) is normal or low: osteopenia if BMD
between -1 and -2.5 standard deviations (SDs) below peak bone
mass of young adult population; osteoporosis if less than -2.5 SDs.
(iv) Given that the patient requires a general anaesthetic for fixation
of the fracture, an opportunistic bone biopsy would be very helpful,
especially in securing the diagnosis of a malignant cause.

28
CALCIUM AND BONE DISEASE CASE 15

A 37-year-old man with chronic renal failure due to IgA nephropathy is


screened for biochemical evidence of renal osteodystrophy.

Na+ 139 mmol/L (135-145)


K+ 4.2 mmol/L (3.4-4.9)
Urea 17.8 mmol/L (2.5-8.0)
Creatinine 349 µmol/L (40-130)

Total Calcium 1.85 mmol/L (2.20-2.60)


Adj Calcium 1.99 mmol/L (2.20-2.60)
Phosphate 1.90 mmol/L (0.7-1.4)
Albumin 40 g/L (40-52)
Alk Phos 350 U/L (80-280)
PTH 26.2 pmol/L (1.0-5.0)

Explain these results.

29
CALCIUM AND BONE DISEASE ANSWER 15

The increased urea, creatinine and phosphate are consistent with reduced
glomerular function. The bone biochemistry is abnormal, with low
albumin-adjusted calcium associated with raised alkaline phosphatase and
parathyroid hormone (PTH).

The hypocalcaemia of chronic kidney disease is multifactorial. First,


reduced glomerular filtration leads to hyperphosphataemia, which results
in hypocalcaemia by enhancing calcium/phosphate complexes. Second,
destruction of nephrons leads to reduced renal hydroxylation of vitamin D
(i.e. reduced formation of 1, 25-(OH)2-vitamin D from 25-(OH)-vitamin
D), as well as reduced absorption of calcium from the gut, and this will
result in hypocalcaemia. Hyperphosphataemia will also inhibit the above
hydroxylation. The parathyroids compensate by secreting PTH
(secondary, or appropriate hyperparathyroidism); this is not cleared as
rapidly as normal for the same reason (reduced glomerular filtration).

30
CALCIUM AND BONE DISEASE CASE 16

A 78-year-old housebound man is seen at a routine visit by his GP. On


review, he complains of facial paraesthesiae, and is found to have marked
carpopedal spasm. The GP sends urgent bloods to the laboratory, the
results of which are shown below.

Total Calcium 1.00 mmol/L (2.20-2.60)


Adj Calcium 1.20 mmol/L (2.20-2.60)
Albumin 37 g/L (40-52)
Alk Phos 170 U/L (80-280)

(a) Give a differential diagnosis for severe hypocalcaemia.

(b) Name and describe two eponymous clinical signs associated with this
electrolyte abnormality.

(c) What additional biochemical investigations may clarify the cause?

31
CALCIUM AND BONE DISEASE ANSWER 16

(a) In a 78-year-old man the most likely potential causes of hypocalcaemia


would be chronic renal failure and/or vitamin D deficiency. Other
causes to be excluded include hypomagnesaemia and
hypoparathyroidism. Rarer still is pseudohypoparathyroidism, a
congenital disorder characterised by tissue resistance to the actions of
PTH (clinically look for short fourth and fifth metacarpals). Post-
operative, acquired (iatrogenic), hypo-parathyroidism following neck
surgery is not a consideration here.
(b) The Chvostek sign is one of the signs of tetany seen in hypocalcemia.
It refers to an abnormal reaction to stimulation of the facial nerve.
When the facial nerve is tapped at the angle of the jaw, the facial
muscles on the same side of the face twitch momentarily because of
hypocalcaemia and resultant hyperexcitable nerves.
Trousseau’s sign of latent tetany is sometimes seen with hypocalcaemia.
This sign is thought to be more sensitive than the Chvostek sign. To
elicit the sign, a blood pressure cuff is inflated to greater than systolic
blood pressure for three minutes, occluding the brachial artery. In the
absence of blood flow, hypocalcemia and consequent neuromuscular
irritability induces spasm in the muscles of the hand and forearm,
resulting in main d'accoucheur ("hand of the obstetrician"), so-called
because it supposedly resembles the position of an obstetrician's hand
in delivering a baby.
(c) Initially, phosphate (if not already requested in the bone profile) and
magnesium should be measured. Phosphate should be low or low-
normal where secondary hyperparathyroidism has resulted from
vitamin D deficiency, and high or high-normal in hypoparathyroidism
(one of the actions of PTH is to increase urinary excretion of
phosphate). Magnesium measurement will confirm or exclude it as the
cause of hypocalcaemia; magnesium is required for PTH release so
magnesium deficiency gives an acquired hypoparathyroidism which
resolves on administration of magnesium. Additional investigations,
which are self-explanatory in the light of the above, include vitamin D
and PTH.
32
CALCIUM AND BONE DISEASE CASE 17

A 69-year-old woman is examined at the Chest Clinic following several


episodes of coughing up blood. She complains of abdominal pain,
constipation and progressive weakness and has lost weight in recent
months. Recently, she has been persistently nauseated and has vomited
frequently.

Total Calcium 2.72 mmol/L (2.20-2.60)


Adj Calcium 3.02 mmol/L (2.20-2.60)
Phosphate 0.51 mmol/L (0.7-1.4)
Albumin 32 g/L (40-52)
Alk Phos 350 U/L (80-280)

(a) What further biochemical investigations may be helpful?

(b) What is the likely diagnosis?

33
CALCIUM AND BONE DISEASE ANSWER 17

(a) Note first that the adjusted calcium is elevated; the low albumin
hides the true severity of the hypercalcaemia. To aid in the
differential diagnosis PTH should be requested.

(b) Diagnosis is most likely that of hypercalcaemia of malignancy. If


this is correct, PTH will be undetectable. Note that patients with
hypercalcaemia of malignancy do not necessarily have metastatic
involvement of bone. Perhaps 20% do not have bone secondaries.
It is now known that humoral agents – hormone like factors – are
involved, and are secreted by the tumour. An example is PTH
related peptide (PTHRP) (see answer to Case 14). Actual ectopic
secretion of PTH by tumours is very rare, if indeed it occurs at all.

34
LIVER FUNCTION CASE 18

A 73-year-old woman presents to her doctor with intense generalised itch


and yellow skin. She has lost an unspecified amount of weight. On
questioning, she denies any pain, but admits that her stools are pale. She is
on no medications. On examination she is deeply jaundiced.

Results of liver function tests are as follows:

Bilirubin 275 µmol/L (3-22)


ALT 42 U/L (3-55)
Alk Phos 980 U/L (80-280)
Albumin 36 g/L (40-52)

(a) What is jaundice?

(b) Why did the doctor ask about pale stools? What do they signify?

(c) Which investigations would you do next?

(d) What type of jaundice is this?

35
LIVER FUNCTION ANSWER 18

(a) Jaundice refers to yellow discoloration of the skin and sclerae due to
the deposition of bilirubin. It is also known as icterus (and jaundiced
patients described as icteric). The upper limit of normal for bilirubin
concentration is ~20 µmol/L; jaundice should be clinically evident
when bilirubin rises to 35-50 µmol/L. Depending on the colour of the
patient's skin and the light in which the patient is being examined, it
may be difficult to be sure; most clinicians teach students to examine
formally for jaundice by looking at the sclera where yellow
discoloration is less ambiguous.
(b) It is impossible to understand the pathogenesis of jaundice without a
detailed knowledge of bilirubin metabolism and excretion. Pale stools
result from the absence from the gut of bile pigments (which give
stools their normal colour), due to complete obstruction of the
common bile duct (CBD). They are usually associated with dark urine
(see below).

(c) If you suspect CBD obstruction, you should arrange liver imaging.
Hospital-based ‘jaundice clinics’ often expedite liver ultrasound or
more detailed diagnostic imaging. In the meantime, you could dipstick
the patient’s urine. In CBD obstruction, you would expect to find
bilirubin but not urobilinogen. Bilirubinuria signifies that conjugation
of bilirubin is occurring (conjugated bilirubin is water soluble, and can
be excreted in the urine). The absence of urobilinogen from the urine
signifies that no bilirubin is reaching the gut, i.e. there is an
obstruction to the flow of bile (see above). Bile pigments normally
leave the gut in an enterohepatic circulation, and are water soluble. In
the gut they are called stercobilinogen, and in the urine urobilinogen.
(d) Obstructive (‘surgical’, post-hepatic) jaundice. Characterised by raised
bilirubin, alkaline phosphatase and gamma glutamyl transpeptidase
activities, with relatively normal transaminase activities.

36
LIVER FUNCTION CASE 19

A 16-year-old schoolboy attends his general practitioner with a history of


excessive fatigue and respiratory symptoms; a lower respiratory tract
infection is diagnosed and antibiotics prescribed. At his initial visit, the GP
orders investigations, including liver function tests, the results of which are
shown below.

Bilirubin 34 µmol/L (3-22)


ALT 20 U/L (3-55)
Alk Phos 135 U/L (80-280)
Albumin 47 g/L (40-52)

(a) Comment on these results. What is the likely diagnosis?

(b) How would you confirm this?

(c) What is the prognosis in this condition?

37
LIVER FUNCTION ANSWER 19

(a) There is no evidence of liver damage or reduced liver synthetic


function. There is an isolated rise in bilirubin. The most likely
diagnosis is Gilbert’s syndrome. This is an inherited defect in
hepatic uptake of bilirubin due to reduced conjugation, due in turn
to reduced activity of glucuronyltransferase, one of the key enzymes
involved. It is characterised by a rise in bilirubin, which is
unconjugated.

(b) The easiest way to secure the diagnosis of Gilbert’s syndrome is by


molecular genetic testing; the commoner mutations responsible are
well recognised. If this is not available, then showing that the rise is
exclusively in unconjugated bilirubin is consistent. Previously, it was
sometimes recommended to reduce energy intake to 400 cal/day for
24-72 hours, and measure bilirubin pre and post; unconjugated
bilirubin should at least double in Gilbert's syndrome (in normal
individuals it does not rise above 25 µmol/L). However this is
virtually never performed today. Nor is the nicotinic acid test
(intravenous injection of nicotinic acid 50 mg with serial
measurement of plasma unconjugated bilirubin over 3 hours).

(c) Individuals with Gilbert's syndrome have a normal life expectancy.


Bilirubin concentration rarely rises above 50 µmol/L and virtually
never over 85 µmol/L. Affected subjects should be reassured and
prognosis regarded as entirely normal for life insurance purposes.
This is the main reason to establish the diagnosis. Jaundice may be
precipitated by fasting, stress, exertion or infections.

38
LIVER FUNCTION CASE 20

A 37-year-old woman is admitted the morning after an overdose. It was


unclear exactly what she took, or when. There was an extensive history of
self-harm, with several previous admissions for paracetamol overdose,
some of which were associated with alcohol. Admission bloods included
liver function tests, paracetamol and salicylate levels, the results of which
are shown below.

Bilirubin 21 µmol/L (3-22)


ALT 2700 U/L (3-55)
Alk Phos 190 U/L (80-280)
GGT 1265 U/L (80-280)
Albumin 41 g/L (40-52)

Paracetamol 76 mg/L
Salicylate Not Detected

(a) Summarise the main findings from these results.

(b) How would you decide whether or not to treat this paracetamol
overdose?

(c) What is the significance of the GGT result?

39
LIVER FUNCTION ANSWER 20

(a) First, ALT is grossly elevated, indicating severe hepatic necrosis (liver
damage). Second, GGT is markedly elevated as well. Third,
paracetamol is clearly detectable. All of these findings should ring
alarm bells, suggesting as they do that this patient has taken a
significant overdose of paracetamol that has already resulted in serious
liver damage.
(b) The paracetamol concentration should be interpreted in conjunction
with the nomogram, a plot of paracetamol concentration against time
in hours. The British National Formulary nomogram shows
paracetamol on two ‘y’ axes, each in different units. ALWAYS, but
ALWAYS, quote (or ask for) the units in which paracetamol has been
measured. Failure to do so can result in a fatal decision to withhold
treatment. In this case, you do not know the time of the overdose so it
is impossible to plot a specific point on the nomogram. However, the
paracetamol result means that you would have to be sure that the
overdose was taken within the previous four hours in order
confidently to withhold treatment. You cannot be sure of that here
and therefore should treat, with intravenous acetylcysteine.
(c) The fact that GGT is clearly elevated, but not alkaline phosphatase or
bilirubin, suggests enzyme induction rather than biliary obstruction or
pathology; this has an important bearing on the threshold for
treatment. The nomogram has two separate treatment curves, one of
which is labelled ‘normal’, the other ‘high-risk’. High-risk patients
include: (i) those on enzyme-inducers like anticonvulsants, or alcohol;
(ii) those who are chronically malnourished (e.g. anorexic patients) or
who have not eaten for a few days. (Alcoholic patients often fit into
both of these categories and so are at especially high risk). As you can
see from the position of the two lines, the threshold for treating high-
risk patients is lower. Enzyme induction is important because
paracetamol only gives rise to toxicity when it is metabolised to toxic
metabolites; enzyme induction speeds this dangerous process up.

40
LIVER FUNCTION CASE 21

A 64-year-old woman presents with a three-month history of feeling very


tired, and generalised aches and pains. Her weight is stable, and there are
no localising symptoms. There is a past medical history of mastectomy for
breast carcinoma seven years previously.

(a) Give a differential diagnosis.

Investigations include liver function tests, the results of which are shown
below.

Bilirubin 16 µmol/L (3-22)


ALT 47 U/L (3-55)
Alk Phos 710 U/L (80-280)
Albumin 41 g/L (40-52)

(b) Comment on these results. Give a revised differential diagnosis.

(c) What are the most likely tissue sources of alkaline phosphatase?

(d) What investigations might help identify the source?

41
LIVER FUNCTION ANSWER 21

(a) These symptoms are non-specific and common; the differential


diagnosis is extensive. It includes various kinds of infection,
hypothyroidism, connective tissue diseases, and, given the past
medical history, metastatic breast cancer. Reasonable investigations
would include full blood count (looking for raised white cell count
suggestive of infection, or raised lymphocytes in the differential
white cell count suggestive of either viral or tubercular infection),
thyroid function tests (to exclude hypothyroidism), bone profile in
view of aches and pains (looking for e.g. hypercalcaemia, or
biochemical evidence of osteomalacia), urea and electrolytes (fatigue
is a common presentation of renal impairment).
(b) There is an isolated elevation of alkaline phosphatase, but the other
liver function tests are unremarkable. In a patient with a history of
malignancy, this is worrying, suggesting as it does the possibility of
metastatic disease. In the light of these LFT, this is the diagnosis
until proven otherise.
(c) Alkaline phosphatase is found in several tissues, but in the vast
majority of cases, the question being asked is: does the raised
alkaline phosphatase come from liver or bone? Less commonly,
placenta and intestine come into the frame as other potential tissue
sources.
(d) The first, and easiest, way to distinguish bone and liver sources of
alkaline phosphatase is to measure GGT. If normal, then it is
unlikely that the raised alkalkine phosphatase has a liver/biliary
source (both enzymes line the biliary tree and are similarly affected
by most pathologies). Alkaline phosphatase isoenzyme estimation is
theoretically useful, but in practice you would not rely exclusively
on this to decide the tissue source. Most clinicians would request an
isotope bone scan, looking for increased isotope uptake either in
bone or liver.

42
THYROID FUNCTION CASE 22

A 62-year-old woman attends her GP complaining of feeling tired all the


time. She also complains of significant weight gain, but denies any change
in lifestyle. On questioning she admits to feeling cold. Physical
examination is unremarkable and no goitre is detected.

(a) Give a differential diagnosis.

(b) What investigations would be appropriate?

43
THYROID FUNCTION ANSWER 22

(a) The differential diagnosis is extensive; comparatively common causes


of her symptoms include hypothyroidism, anaemia, depression and
chronic renal disease.

(b) Her thyroid function tests, full blood count and urea and electrolytes
should be measured.

FBC and U&E are unremarkable. A free T4 of <3 pmol/L and TSH
47 mU/L indicate that she has primary hypothyroidism. The TSH
increases to compensate for the relative absence of the thyroid
hormones. TSH is the most sensitive test for primary
hypothyroidism. Sometimes it is increased when the T4 is normal and
this is sub-clinical hypothyroidism. Thyroid antibodies should be
measured and repeat TFTs performed.

44
THYROID FUNCTION CASE 23

A 33-year-old woman presents with increasing nervousness and 10 kg


unintentional weight loss over the previous 6 months. She is worried about
the possibility that she may have cancer. On questioning, she admits to
fatigue, irritability and paradoxically an increase in appetite.

On examination a tremor is detected; her pulse rate is 100 bpm (sinus


rhythm). The GP checks for thyroid eye disease and finds lid retraction,
but nothing else. He suspects hyperthyroidism.

(a) What thyroid function tests are commonly available?

(b) What are the therapeutic options if the patient is hyperthyroid?

45
THYROID FUNCTION ANSWER 23

(a) Thyroid-stimulating hormone (TSH) is measured as a first-line


thyroid function test in almost every biochemistry laboratory. Many
also measure free thyroxine (fT4) as a first-line test. Most keep T3
(total or free) as a second-line or third-line test (it is sometimes
raised before fT4 in early or developing hyperthyroidism). Primary
hyperthyroidism is the most likely diagnosis here. The biochemical
picture seen in this condition is of increased fT4 resulting in
suppression of TSH. Relevant further questions include family
history of thyroid disease or other autoimmune diseases.

(b) Broadly, the treatment of hyperthyroidism is with one of the


following three modalities: medical therapy (tablets); radio-iodine;
surgery.

Medical therapy is the first line treatment and the choice is between
carbimazole and propylthiouracil. The former is usually preferred,
but in certain cases (e.g. pregnancy) propylthiouracil is the preferred
option. Patients should be warned about skin rash, infection (due
to low white cell counts) and jaundice as potential side effects of the
medical therapy.

Radio-iodine is reserved for patients who cannot tolerate medical


therapy, or where the hyperthyroidism is difficult to control
medically. It causes irreversible hypothyroidism and the patient
requires lifelong T4 replacement thereafter.

Thyroid surgery is usually preferred only if there are pressure


symptoms (dysphagia, stridor and hoarseness) due to a goitre, or if
other modalities of treatment are contraindicated.

46
THYROID FUNCTION CASE 24

A 37-year-old man attends his GP complaining of fatigue. Investigations


include Thyroid Function Tests, the results of which are as follows:

fT4 5.2 pmol/L (9-22)


TSH 3.5 mU/L (0.4-4.0)

(a) Comment on these results.

(b) What other investigations should be performed?

(c) How should this patient be treated?

47
THYROID FUNCTION ANSWER 24

(a) The fT4 is unequivocally low indicating hypothyroidism. However,


the TSH is inappropriately low, indicating that the diagnosis is not
primary hypothyroidism. The possibility of a central cause for his
hypothyroidism, e.g. hypopituitarism, needs to be excluded.

(b) The investigations should be directed towards establishing the


adequacy or otherwise of pituitary reserve of TSH and other
anterior pituitary hormones. The anterior pituitary gland produces
hormones that are trophic for the following axes: thyroid, adrenal,
gonadal, growth. Baseline anterior pituitary function tests should
include cortisol (as a proxy for ACTH, which is unstable and
requires to be collected on ice), LH and FSH, as well as IGF-1. If
the results of these tests are not entirely reassuring, then dynamic
function testing should be performed. The components of a
complete anterior pituitary function test are: (1) insulin stress (or
tolerance) test (insulin-induced hypoglycaemia stresses the pituitary
into producing ACTH and GH; (2) TRH test (exogenous TRH
should stimulate production of TSH; (3) LHRH (exogenous LHRH
should stimulate production of LH and FSH).

In addition to biochemical tests, investigations must also include


pituitary imaging (e.g. MRI); the commonest cause of
hypopituitarism is a pituitary adenoma.

(c) Treatment consists of administering the relevant replacement


hormones. The choice of therapy depends on the extent of the
hypopituitarism, i.e. which axes are affected. Pituitary adenomas
should be resected surgically, sometimes augmented by
radiotherapy. Thereafter replacement hormones should be
prescribed as appropriate.

48
THYROID FUNCTION CASE 25

A 78-year-old woman is admitted to hospital with a left-sided hemiparesis.


As part of her clinical evaluation Thyroid Function Tests are requested.

These showed the following results:

fT4 7.2 pmol/L (9-22)


TSH 2.5 mU/L (0.4-4.0)

(a) Comment on these results.

(b) Does this patient require thyroxine replacement?

49
THYROID FUNCTION ANSWER 25

(a) The fT4 is low but the TSH is in the reference interval. Low fT4
without a compensatory rise in TSH may indicate pituitary (secondary)
hypothyroidism. However, these results may also reflect non-
thyroidal illness. In systemic illness the normal regulation of TSH, T4
and T3 secretion, and the subsequent metabolism of the thyroid
hormones, is disturbed. Thyroid Function Tests should be repeated
once the patient has recovered from the acute phase of her stroke.

(b) It is reasonable to withhold thyroxine replacement therapy pending


this.

50
ADRENAL FUNCTION CASE 26

A 43-year-old obese (BMI 43 kg/m2) hypertensive woman is reviewed by


her GP, who thinks her face looks a bit rounder/puffier than usual and
wonders if she might be Cushingoid. Initial investigations include serum
urea and electrolytes, random plasma cortisol and thyroid function tests,
the results of which are shown below.

Na+ 143 mmol/L (135-145)


K+ 3.2 mmol/L (3.4-4.9)
Urea 6.2 mmol/L (2.5-8.0)
Creatinine 85 µmol/L (40-130)
Cortisol 479 nmol/L (240-720 for samples collected 8-10am)

f T4 14.8 pmol/L (9-22)


TSH 1.6 mU/L (0.4-4.0)

(a) What is meant by the term ‘Cushingoid’?

(b) What do the results above tell you about the likelihood or otherwise
of this patient having Cushing’s syndrome?

(c) What investigations (that can be done in primary care) would help the
GP to decide if further investigations and/or referral are needed?

51
ADRENAL FUNCTION ANSWER 26

(a) ‘Cushingoid’ means having some or all of the features of Cushing’s


syndrome. Cushing’s syndrome is the collection of signs and symptoms
that result from cortisol excess. They include (list is not exhaustive)
moon face, acne, plethoric cheeks, hypertension, thin skin, bruising,
abdominal striae, and dorsal fat pad (‘buffalo hump’). Cushing’s
syndrome is most often caused by corticosteroid medications
prescribed for inflammatory conditions like asthma, arthritis and
inflammatory bowel disease. This patient was not on corticosteroids.

(b) There are pointers in these results, but nothing conclusive. The
mineralocorticoid activity of excess cortisol will tend to drive the renal
reabsorption of sodium ions in exchange for potassium and hydrogen
ions, so the slightly low serum potassium seen here is consistent (but
not diagnostic) with suspected cortisol excess. The random plasma
cortisol result here is not helpful. Cortisol has a circadian rhythm, with
highest concentrations in the earlier part of the day, and is therefore
quite variable. Only very high or very low results are helpful.

(c) The most convenient test for both patient and doctor is the overnight
dexamethasone suppression test. This involves prescription of a single
1mg tablet of dexamethasone, an exogenous corticosteroid that
suppresses secretion of ACTH and through it the cortisol axis. The
patient takes the tablet in the evening, e.g. 10.00 pm and attends the
practice the following morning for a 9.00 am plasma cortisol
measurement. If the cortisol has suppressed to < 50 nmol/L then the
patient’s cortisol axis is deemed to be normal and not requiring further
investigation. Alternatively urinary free cortisol can be measured, either
on a spot sample, or preferably a 24-hour collection (ideally several
sequential samples should be collected). The result is usually expressed
as a ratio of urinary cortisol to creatinine.

52
ADRENAL FUNCTION CASE 27

A 55-year-old Pakistani woman is seen in the endocrine clinic,


complaining of excessive fatigue, and of one or two episodes of
fainting/collapse. She is accompanied by her daughter who interprets for
her mother whose spoken English is limited.

Thyroid disease has already been excluded by the GP who wonders if


there is another endocrine explanation for the fatigue. On examination,
she appears dry. Supine blood pressure is 100/60 mmHg falling to 70/40
mmHg on standing. The patient’s skin is noticeably darker than her
daughter’s.

(a) What is the significance of postural hypotension here?

(b) What investigations should be arranged?

(c) How should this patient be treated?

53
ADRENAL FUNCTION ANSWER 27

(a) Postural hypotension and dehydration should make you think of


sodium depletion. Steroid hormones produced by the adrenal glands
have mineralocorticoid activity, which regulates renal reabsorption of
sodium. If this is reduced, excessive urinary loss of sodium – and water
– occurs with concomitant retention of K+ and H+. The difference in
skin colour of mother and daughter should flag up possible excessive
pigmentation due to primary adrenal insufficiency, although differences
in the amount of time spent in e.g. Pakistan should also be explored.
The mechanism involves excessive anterior pituitary secretion of
ACTH due to lack of negative feedback from adrenal steroids. The
amino acid sequence of ACTH contains the pigmenting melanocyte-
stimulating hormone (MSH), which is released due to the action of
proteases on ACTH in the blood.
(b) Adrenal insufficiency needs to be excluded urgently. Initial
investigations from the clinic should include serum electrolytes,
random plasma cortisol and ACTH. If the cortisol and ACTH are
inconclusive, formal documentation of adrenal insufficiency requires
that a Synacthen test be performed (Synacthen is synthetic ACTH). In
a Pakistani patient, tuberculous infiltration of the adrenal glands is a
real possibility, although auto-immune destruction should also be
considered. In this patient, random cortisol was 30 nmol/L (very low);
ACTH 120 ng/L (very high), indicating primary adrenal insufficiency.
(c) This patient requires intravenous fluid resuscitation with isotonic
(0.9%) saline (to replace the sodium she has lost), and steroid
replacement. Initially this may have to be given IV (if patient is too ill
to take oral steroid replacement). Thereafter she can be converted to
oral therapy. A typical regimen for oral steroid replacement is 20 mg
hydrocortisone in the morning and 10 mg in the evening, often with
100 or 200 µg of fludrocortisone in the morning to provide the
required mineralocorticoid activity. The Synacthen test can be done
routinely in due course (delaying the morning doses of hydrocortisone
and fludrocortisone which may interfere with the cortisol assay).

54
ADRENAL FUNCTION CASE 28

An 18-year-old woman is referred for investigation of distressing


hirsutism. She shaves daily and has noticed deepening of her voice and
scalp hair loss. On questioning she admits to irregular menstrual periods.
On examination she is clearly hirsute with male-pattern distribution of
body hair; temporal balding and deep voice are confirmed.

(a) Give a differential diagnosis for androgen excess in an adult female


patient.

(b) What investigations would help you to establish the diagnosis?

(c) The results of a Synacthen test are shown below. Interpret the results.

Cortisol 17 alpha-hydroxy progesterone


(nmol/L) (nmol/L)

0 minutes 274 15
30 minutes 468 33
60 minutes 513 85

55
ADRENAL FUNCTION ANSWER 28

(a) The differential diagnosis of androgen excess includes an androgen-


secreting tumour (usually of adrenal or ovarian origin), late-onset
congenital adrenal hyperplasia, and polycystic ovarian syndrome. The
latter is much commoner than the other two, but is not associated with
virilisation, as here, which should prompt a concerted search for the
source of androgen.

(b) Imaging of the adrenals and ovaries should be performed. Testosterone


and SHBG should be measured as a minimum biochemical evaluation
of androgen status; additional adrenal androgens which may be helpful
include androstenedione and DHA sulphate. Synacthen test with
measurement of 17 alpha-hydroxy-progesterone should be done to
investigate congenital adrenal hyperplasia.

(c) In late-onset congenital adrenal hyperplasia (CAH), concentrations of


17 alpha-hydroxy-progesterone are not as high as in ‘classical’ CAH
where the enzyme deficiency is more severe and the presentation much
earlier in life. However, they are usually higher than in normal
individuals and can be more clearly differentiated from normality
following stimulation by Synacthen. As a rule of thumb, 17 alpha-
hydroxy-progesterone concentrations >50 nmol/L at 60 minutes (as
here) are diagnostic of late-onset CAH.

56
ADRENAL FUNCTION CASE 29

A 41-year-old man is referred for specialist advice regarding management


of hypertension.

Blood pressure is reported by his general practitioner to be 160/90 mmHg


on triple therapy of ramipril 5mg, amlodipine 10mg and bendrofluazide
2.5mg. Ambulatory blood pressure monitoring demonstrates variable
readings, but overall confirms uncontrolled hypertension (mean daytime
readings 152/88 mmHg). The patient gives a history of panic attacks, each
lasting a few minutes, associated with severe sweating and palpitation.

(a) What secondary causes of hypertension should be excluded routinely?

(b) What investigations may help in this process?

57
ADRENAL FUNCTION ANSWER 29

(a) Phaeochromocytoma and renal artery stenosis should routinely be


screened for. The episodic nature of his symptoms (panic attacks), and
the variability in blood pressure during 24-hour monitoring, is
characteristic of phaeochromocytoma and this condition must be
excluded. Renal artery stenosis is more likely in a patient with
established vascular (especially peripheral vascular) disease. Other
endocrine causes of secondary hypertension include diabetes,
acromegaly, primary hyperaldosteronism and cortisol excess (Cushing’s
syndrome).

(b) Routine investigations should include U&E and glucose (preferably


fasting). Urinary excretion of catecholamines (three separate 24-hour
collections into acidified containers) is the usual screening test for
suspected phaeochromocytoma, but false negatives and false positives
are common; plasma metanephrines are the most sensitive biochemical
investigation available. Imaging of the adrenals should be performed
and the finding of a mass may prompt surgical exploration and
removal.

Other investigations include magnetic resonance angiography of renal


arteries (looking for renal artery stenosis), and MIBG isotope scan
(phaeochromcytoma), and measurement of plasma renin and
aldosterone (primary hyperaldosteronism), especially in the presence of
a low serum potassium.

58
DIABETES CASE 30

A 43-year-old man presents with symptoms of thirst, nocturia and a lack


of energy. As part of his investigation his GP performs a fasting blood
glucose and result is 7.6 mmol/L.

(a) How would you interpret the glucose result?

(b) What would you do next?

59
DIABETES ANSWER 30

(a) The glucose result places him in the diabetic category (≥7.0 mmol/L).

(b) Diabetes is usually a lifelong diagnosis and most professional bodies


recommend that it should not be based solely on the result of a single
blood test. According to current guidelines, this result should therefore
be followed up by a repeat fasting glucose, either alone or as part of an
oral glucose tolerance test (OGTT).

The diagnosis of diabetes is currently under review globally. The role


of the OGTT in particular is increasingly marginal. Two-hour glucose
results are difficult or impossible to interpret if the patient has not
rested throughout the test. Glycated haemoglobin, which continues to
be used in the monitoring of diabetes, may also be used to diagnose
diabetes, and does not require the patient to be fasting.

The options are:

i. repeat fasting glucose


ii. oral glucose tolerance test
iii. glycated haemoglobin.

60
DIABETES CASE 31

A 63-year-old woman with a recent history of oral thrush is being


investigated to determine its aetiology. Previous fasting glucose was
recorded as 6.8 mmol/L. An oral glucose tolerance test is performed to
establish her glycaemic status.

Fasting glucose 8.9 mmol/L; 2-hour glucose 12.3 mmol/L.

How would you interpret:

(a) the original fasting glucose result?

(b) the results of the oral glucose tolerance test?

61
DIABETES ANSWER 31

(a) The original fasting glucose puts this patient in the category of
impaired fasting glycaemia (IFG) (6.0-6.9 mmol/L). As the label
suggests, this is an intermediate category between normal and diabetes.
It arose out of a decision made in 1997 by the American Diabetes
Association to recommend the abolition of the oral glucose tolerance
test, which is required for the diagnosis of impaired glucose tolerance
(IGT – see below); IFG was meant to be a glycaemic category roughly
equivalent to IGT, although the evidence for equivalence is limited.
Patients with IFG and IGT are at risk of developing full-blown
diabetes, and also at elevated cardiovascular risk; they should undergo
periodic screening for diabetes, and their cardiovascular risk factors
should be managed aggressively.

(b) Both the fasting and 2-hour results place this patient in the diabetic
category. The respective criteria are ≥7 mmol/L (fasting) or ≥11.1
mmol/L (2 hour). A 2-hour glucose between 7.8 mmol/L and 11.0
mmol/L would place the patient in the intermediate category of
impaired glucose tolerance, a pre-diabetic state conferring increased
cardiovascular risk.

62
DIABETES CASE 32

A 38-year-old man with acromegaly is treated initially with surgery and


radiotherapy. Subsequently his original symptoms of arthralgia, fatigue and
sweating recur. Further investigations include an oral glucose tolerance
test. The results are shown below.

Fasting glucose: 6.8 mmol/L; 2-hour glucose: 10.3 mmol/L

(a) Why was this oral glucose tolerance test performed?

(b) What else would you measure during the procedure?

(c) How would you interpret these glucose results?

63
DIABETES ANSWER 32

(a) The oral glucose tolerance test was performed for two reasons: first, to
see if his acromegaly had recurred; second, to assess his diabetic status.
(You could answer the second question, but not the first, by a fasting
glucose alone).

(b) The oral glucose tolerance test is performed with growth hormone
measurement in addition to glucose at each time point as the diagnostic
dynamic function test for acromegaly. In acromegaly there is a failure
of growth hormone to suppress below 1 µg/L; in fact there may be a
paradoxical rise. Patients with acromegaly are at elevated risk of
diabetes (growth hormone is, along with cortisol and adrenaline, a
stress hormone which makes people insulin resistant).

(c) The fasting glucose puts the patient in the IFG category, and the 2-
hour glucose is diagnostic of impaired glucose tolerance (7.8-11.0
mmol/L). So this patient is not diabetic, and his glycaemic status may
improve with successful treatment of his acromegaly.

64
DIABETES CASE 33

A 19-year-old woman presents to A&E with a 3-day history of nausea and


vomiting. She has deep sighing respiration and her breath smells sweet.
The receiving physician makes a clinical diagnosis, arranges investigations,
and commences her on treatment.

(a) What is the clinical diagnosis?

(b) What investigations are appropriate to confirm this diagnosis?

(c) What are the principles of treatment?

65
DIABETES ANSWER 33

(a) The clinical diagnosis is diabetic ketoacidosis (DKA). The respiration is


characteristic of severe metabolic acidosis and indeed is known as
acidotic (or Kussmaul) breathing. It is not specific to ketoacidosis or
diabetes; the clinical diagnosis was based also on the associated finding
of sweet-smelling breath, due to the presence of volatile ketone bodies.
These are the acidic products of fatty acid breakdown, which results
from the acute need for alternative fuel sources when glucose entry
into cells becomes critically reduced. Ketoacidosis can also occur in the
absence of diabetes, particularly in alcoholism, but it is usually clear
from the clinical context whether this alternative diagnosis should be
considered.

(b) The most pressing relevant investigations are: arterial blood gases;
blood glucose; urea & electrolytes. Plasma glucose is very high (often
>40 mmol/L), along with raised urea and creatinine, consistent with
hypovolaemia. Arterial blood gases allow the acid-base status to be
assessed. DKA is usually associated with a severe metabolic acidosis
(i.e. serum bicarbonate is very low, reflecting the fact that it has been
‘mopped up’ by the acid load resulting from ketone body formation).
The PCO2 is usually also reduced, reflecting acute respiratory
compensation for the metabolic acidosis. This compensation is not
usually enough to restore acidaemia to normal, so [H+] is usually
increased. Thus, a partially compensated severe metabolic acidosis is
characteristic.

(c) The key principles of treatment are: (i) to lower blood glucose by
administering insulin; (ii) to replace fluids (patients will be fluid
depleted, often severely, due to osmotic diuresis induced by
hyperglycaemia, and vomiting); (iii) to maintain potassium balance
(potassium will move into cells as well as glucose in response to
insulin); (iv) to correct the acid-base status (this is usually achieved by
means of the above; bicarbonate is rarely given except where acidaemia
is extreme).

66
DIABETES CASE 34

An 81-year-old man with type 2 diabetes is brought to A&E from the


residential home where he lives. He has been unwell for several days,
culminating in nausea and vomiting on the day of admission. On
examination he is confused and dehydrated. Point-of-care glucose is 57
mmol/L. Breathing is unremarkable and his breath does not smell ketotic.
He is unable to give a urine sample, but his serum tests negative for
ketones in the hospital biochemistry department. Blood results are shown
below.

Na+ 149 mmol/L (135-145)


K+ 4.2 mmol/L (3.4-4.9)
Urea 22.3 mmol/L (2.5-8.0)
Creatinine 245 µmol/L (40-130)
eGFR 22 mL/min/1.73m2
HCO3- 24 mmol/L (21-28)
Glucose 65 mmol/L (4.0-5.5)

Arterial Blood Gases


H+ 44 nmol/L (35-45)
PCO2 5.7 kPa (4.5-5.6)
PO2 11.3 kPa (12-15)

(a) Comment on the results.

(b) What is the diagnosis?

(c) Why do some patients with diabetes develop DKA and others this
complication?

67
DIABETES ANSWER 34

(a) The most striking abnormality is his plasma glucose. Other biochemical
abnormalities are hypernatraemia and clear evidence of reduced
glomerular function (urea and creatinine both markedly elevated). Urea
is disproportionately elevated in relation to creatinine. This and the
hypernatraemia is suggestive of dehydration. The arterial blood gas
results are notable chiefly because they provide little evidence of
acidaemia, or a metabolic acidosis (which might reasonably be expected
in the clinical context of gross hyperglycaemia).

(b) The diagnosis is hyperosmolar non-ketotic coma (HONK). The ease


of remembering the acronym helps to explain the persistence of this
rather misleading term (some prefer non-ketotic acute hyperglycaemia).
The term HONK is misleading on a couple of grounds: first,
comparatively few patients with this condition are comatose; second,
although it is indeed a hyperosmolar state, so is DKA. The critical
distinction between HONK and DKA is the absence of ketone bodies
and associated acidosis and acid-base disturbance.

(c) As pointed out above, the critical distinction between HONK and
DKA is the absence of ketone bodies in HONK despite severe
hyperglycaemia. This illustrates the fact that insulin resistance, a key
component of the pathogenesis of both complications, is complex. In
both, resistance to insulin-stimulated glucose uptake is present.
However, the relative resistance to another of insulin’s actions
(suppressing lipolysis and release of fatty acids from adipose tissue) is
different in these two states. In DKA the ability of insulin to suppress
lipolysis is grossly reduced, whereas in HONK, it is preserved enough
to prevent the formation of ketone bodies, the intermediates in fatty
acid breakdown.

68
PITUITARY FUNCTION CASE 35

A 45-year-old woman, who had been suffering from oligomenorrhoea for a


few months, went to her GP complaining of headaches and blurring of
vision. He thought her symptoms might be due to early menopause, but
did a fundoscopy due to the visual symptoms and noted that the fundus
looked pale. He measured her prolactin and gonadotrophins along with
routine blood tests. The prolactin result came back as 2500 mU/L. She
was referred to an Endocrinologist and underwent dynamic function testing
to assess the pituitary reserve. She underwent a combined anterior pituitary
test (insulin stress test + TRH Test + LHRH Test). The results (including
the basal reference ranges) were as follows:

Time Glucose Cortisol GH IGF1 PRL FSH LH E2 TSH fT4


mins mmol/L nmol/L µg/L µg/L mU/L U/L U/L pmol/L mU/L pmol/L
(4.0-5.5) (280- (<1) (35- (<630) (3-13) (2-15) (110- (<0.5) (10-22)
720) 240) 700)

0 5.4 318 0.7 25 2718 <0.5 <0.5 <70 2.3 16


30 1.1 410 0.9 2950 <0.5 <0.5 18.1
60 2.1 580 1.4 3025 <0.5 <0.5 11.2
90 3.4 610 1.1
120 4.1 540 1.1

(a) What is the diagnosis?

(b) Comment on the results of the dynamic function tests.

(c) What are the treatment options in this case?

69
PITUITARY FUNCTION ANSWER 35

(a) The history provided and the prolactin results strongly suggest that the
patient has a prolactin secreting tumour. Prolactinomas are classified
according to their size into macro-prolactinomas (>3mm) or micro-
prolactinomas (<3mm). The indications here are that this patient has
a macro-prolactinoma that is compressing adjacent structures
including the optic chiasm, thereby giving rise to visual disturbances
and abnormal fundoscopy. It might reasonably be anticipated,
therefore, that such a large tumour would also be affecting adjacent
pituitary tissue and function.

(b) Insulin-induced hypoglycaemia is the stress stimulus to induce


secretion of cortisol and growth hormone; adequate stimulus is
confirmed by the serum glucose <2 mmol/L. TRH and LHRH
stimulate the secretion of TSH and gonadotrophins, respectively.
Cortisol response of >550 nmol/L and increase of TSH secretion (2-5
times increase over baseline) confirm normal pituitary-adrenal and
pituitary-thyroid axes, respectively. There is a ‘flat’ response for
growth hormone and gonadotrophins; baseline IGF-1 and oestradiol
are also low. This is consistent with severely impaired growth
hormone and gonadotrophin reserve.

(c) Broadly, the treatment options are medical and surgical. Medical
treatment of prolactinomas is based on the principle of inhibiting the
release of prolactin by stimulating dopamine receptors in the
hypothalamus. Dopaminergic drugs reduce serum prolactin and
tumour size and are often prescribed as an adjunct to more definitive
neurosurgical treatment (hypophysectomy). Patients undergoing
hypophysectomy should have their pituitary reserve evaluated both pre
and post operatively.

70
PITUITARY FUNCTION CASE 36

A 40-year-old fork-lift truck driver presented with complaints of multiple


joint pains and lethargy. He had also noticed that his work helmet was not
fitting very well and he had to loosen the chin strap significantly in order to
buckle it in place. He underwent an oral GTT.

Time Glucose GH IGF1


mins mmol/L µg/L µg/L
(4.0-5.5) (<1) (50-315)
0 7.8 7.3 625
30 8.9 6.6
60 12.1 6.1
90 14.6 7.0
120 13.7 6.4

(a) What is the diagnosis and what other clinical features may be present?

(b) Comment on the oral GTT.

The patient underwent surgery and four weeks later undergoes repeat
dynamic function testing.

Time Glucose Cortisol GH IGF1 PRL FSH LH Testost. TSH fT4


mins mmol/L nmol/L µg/L µg/L mU/L U/L U/L nmol/L mU/L pmol/L
(4.0-5.5) (280- (<1) (50- (<400) (3-13) (2-15) (10-35) (0.4- (10-22)
720) 315) 4.0)

0 7.4 110 1.8 298 210 <0.5 <0.5 8 1.6 14


30 8.6 145 3.2 295 <0.5 <0.5 8.5
60 12.3 157 4.8 282 <0.5 <0.5 4.9
90 11.6 144 2.1
120 10.9 112 1.6

(c) What test has been performed and comment on the results?

(d) What short term and long term therapy is required?

71
PITUITARY FUNCTION ANSWER 36

(a) Acromegaly, due to excess growth hormone secretion, usually due to a


pituitary adenoma. (Growth hormone excess before bone growth
stops may give rise to gigantism). Musculoskeletal clinical features are
due to soft tissue and bony overgrowth resulting in enlargement of the
hands and feet, skin thickening, expansion of the skull, protrusion of
the jaw and carpal tunnel syndrome. The adenoma itself can cause
compression of the optic chiasm leading to visual field defects and
neurological features of a space occupying lesion in the cranium.
Systemic complications include diabetes mellitus, hypertension, heart
failure and renal failure.

Baseline growth hormone (and IGF-1) is elevated and there is no


suppression of growth hormone by glucose (suppression below 1 µg/L
effectively rules out acromegaly); in fact there is a paradoxical increase.
Also note that the elevated fasting and 2-hour glucose confirm
diabetes mellitus.

(b) Combined anterior pituitary function test. As is commonly the case


post surgery there is only partial correction of the growth hormone
excess; glucose is still in borderline diabetic range. There is post-
surgical partial hypopituitarism – no cortisol response; very low
gonadotrophins and testosterone.
(c) Most important is corticosteroid replacement therapy as this patient
has very little adrenal reserve and is at risk of cardiovascular collapse
even with modest stress (e.g. common cold). Many patients will also
require testosterone supplements. DDAVP is also required for a few
weeks after surgery for transient diabetes insipidus due to surgical
damage to the hypothalamus. If polyuria persists after stopping
DDAVP, a formal water deprivation test would be performed to
assess for more permanent damage. These patients have to be
followed up long term for recurrence of acromegaly and additional
medical management may be indicated.

72
LIPIDS CASE 37

A 52-year-old man visits his GP for the first time in 25 years with
concerns about his risk of cardiovascular disease. He reports that his 56-
year-old brother has just died suddenly from a myocardial infarction.
Although anxious he is otherwise well with no personal history of
cardiovascular disease.

As part of his cardiovascular risk assessment the GP decides to measure a


lipid profile on a non-fasting sample. The results are shown below:

Total Cholesterol 6.3 mmol/L


HDL-Cholesterol 0.9 mmol/L
Triglyceride 1.3 mmol/L

(a) Comment on the patient’s lipid profile.

(b) What other factors should be taken into account to assess the
patient’s overall cardiovascular risk?

(c) Should this man be prescribed lipid lowering therapy?

73
LIPIDS ANSWER 37

(a) This patient’s total cholesterol is elevated and his HDL-cholesterol


lower than desirable. His triglyceride, although measured on a non-
fasting sample, is not elevated.

(b) While the lipid profile is important, other factors must be taken into
account to assess global cardiovascular risk. These factors vary from
one risk assessment method to another, but routinely will include age,
gender, smoking status, diabetic status, blood pressure, and personal
and family history of cardiovascular disease. The most widely used
method to assess risk is based on the Framingham Risk Equation,
which requires the input of age, gender, total and HDL-cholesterol,
systolic blood pressure and smoking status. This man was a non-
smoker and had a SBP of 140 mmHg and his calculated 10-year risk
of cardiovascular disease is 11%.

(c) Although this man’s total cholesterol level is higher than desirable, we
should manage our patients on the basis of their cardiovascular risk
rather than their absolute lipid values. His calculated risk is only
11%, however he has presented because of a new family history of
cardiovascular disease, which is not taken into account by the
Framingham risk equation. Nevertheless, this patient is still a
relatively low risk primary prevention candidate – i.e. we are trying to
prevent a first event. His risk should be explained and he should be
given appropriate lifestyle advice in terms of both diet and activity.
Improvements in these factors may have a beneficial impact on his
lipid profile and his subsequent cardiovascular disease risk. If lipid
lowering therapy is contemplated it should be remembered that this is
lifelong therapy and the decision to start should not be taken on the
basis of a single lipid profile.

74
LIPIDS CASE 38

A 61-year-old woman is reviewed by her GP 3 months after suffering an


ST elevation myocardial infarction (STEMI). She is prescribed
bendroflumethiazide 2.5mg, ramipril 10mg and simvastatin 20mg. Her
blood pressure is 138/74 mmHg, she has successfully stopped smoking
since her STEMI and her BMI is 26.6 kg/m2.

Her fasting lipid profile at this visit is:

Total Cholesterol 5.9 mmol/L


HDL-Cholesterol 1.1 mmol/L
Triglyceride 3.4 mmol/L

(a) Comment on the patient’s lipid profile.

(b) How would you calculate this woman’s cardiovascular risk?

(c) Should the patient’s lipid lowering therapy be changed?

75
LIPIDS ANSWER 38

(a) This patient’s total cholesterol is elevated despite being on


simvastatin therapy. Her HDL-cholesterol is just at the acceptable
level for a woman, but her triglyceride is elevated.

(b) This woman has already suffered a cardiovascular event. As such she
is now a candidate for secondary prevention – i.e. the prevention of
second or subsequent events. The risk scoring systems we use to
assess overall cardiovascular risk only apply in primary prevention.
This is simply because there is no need to apply complex risk
algorithms to patients with established cardiovascular disease as their
risk is automatically high and merits intervention.

(c) When a patient is not at the desired lipid target despite being on
therapy we must first confirm that they are taking their medication as
prescribed. Compliance with all lipid lowering therapies is poor
largely because apart from measured changed in the lipid profile the
patient sees no tangible benefit especially if their physician has not
explained the purpose of the therapy – i.e. as an insurance policy
against future events. If compliance is an issue this should addressed
before considering any change to the regimen. If compliance is not a
problem the main options are to increase the dose of simvastatin to
40mg or to switch to a different more powerful statin such as
atorvastatin or rosuvastatin. In view of the patient’s triglyceride,
which will only be modestly reduced by statin therapy, other lipid
lowering regimens may be considered.

76
LIPIDS CASE 39

A 48-year-old man is diagnosed with type 2 diabetes, and is seen at the


hospital risk factor clinic. His BMI is calculated as 32.8 kg/m2 and his
blood pressure is 149/96 mmHg. He is a non-smoker and has no family
or personal history of premature cardiovascular disease.

His fasting lipid profile at this visit is:

Total Cholesterol 4.6 mmol/L


HDL-Cholesterol 0.6 mmol/L
Triglyceride 4.9 mmol/L

(a) Comment on the patient’s lipid profile.

(b) How would you calculate this man’s cardiovascular risk?

(c) Should the patient be prescribed lipid lowering therapy?

77
LIPIDS ANSWER 39

(a) The patient has significant hypertriglyceridaemia and an


accompanying low level of HDL-cholesterol. His total cholesterol
level is acceptable.

(b) As with patients who have already suffered a cardiovascular event,


patients with diabetes are considered to be at automatically high risk
when diagnosed. Many regard such patients as the equivalent in
cardiovascular disease risk terms as secondary prevention patients
and therefore no further risk assessment is required.

(c) This man’s cardiovascular disease risk is significant and merits early
intervention in a number of areas. He is overweight, which may be
contributing to his glucose intolerance, his aberrant lipid profile and
his hypertension. Efforts should be made to convince this man of
his cardiovascular disease risk and the need for weight loss. Despite
this he is likely to require anti-hypertensive therapy and, particularly
in view of his type 2 diabetes, he would be considered a candidate
for lipid lowering therapy. Although fibrate therapy might seem the
most obvious lipid lowering choice in view of the prime action of
this class of drugs on triglyceride and HDL, statins rather than
fibrates would be recommended in view of the overwhelming
evidence of their clinical efficacy in reducing cardiovascular risk in
patients with diabetes.

78
TOXICOLOGY CASE 40

A confused 42-year-old man presents to A&E claiming to have taken


‘hundreds’ of aspirin tablets earlier. He is agitated and hyperventilating
(respiratory rate 30 per minute). Glasgow Coma Scale is 15/15. He is
sweating profusely. Blood is taken for salicylate (and paracetamol), arterial
blood gases, electrolytes (including bicarbonate) and glucose. Results are as
follows:

Na+ 138 mmol/L (135-145)


K+ 3.2 mmol/L (3.4-4.9)
Urea 10.4 mmol/L (2.5-8.0)
Creatinine 116 µmol/L (40-130)
HCO3- 18 mmol/L (21-28)
Glucose 2.8 nmol/L (4.0-5.5)

Salicylate 720 mg/L (Upper therapeutic limit 300mg/L)


Paracetamol Not detected

Arterial blood gases

H+ 39 nmol/L (35-45)
PCO2 3.6 kPa (4.5-5.6)
PO2 13.3 kPa (12-15)

(a) Comment on these results.

(b) Grade the severity of this salicylate overdose, justifying your answer.

(c) What are the treatment options?

79
TOXICOLOGY ANSWER 40

(a) U&E, glucose: marked metabolic acidosis (low venous bicarbonate),


hypoglycaemia, mild hypokalaemia, urea slightly elevated. Salicylate
concentration clearly in toxic range. Paracetamol not detected. You
might reasonably wonder why paracetamol was requested, given that
the reported overdose is specifically of aspirin tablets. In many
hospitals, they are requested together routinely, on the twin grounds
that (i) overdose patients may not give a reliable history, and (ii)
salicylate and paracetamol are widely available over-the-counter
medications.
Arterial blood gases: [H+] is normal. Bicarbonate is low (metabolic
acidosis). PCO2 is low as well (respiratory alkalosis). The question is,
which is compensating for the other? Or are they occurring more or
less independently of each other (mixed acid-base disturbance)? The
history of salicylate overdose is critical here. Salicylate stimulates the
respiratory centre in the brain causing hyperventilation and alkalosis.
This normally predominates initially, giving way to a metabolic
acidosis (due to uncoupling of oxidative phosphorylation), although
this is not always the case. The likelihood is that this is a mixed acid-
base disturbance, i.e. that both processes are ‘primary’.
(b) Clinical presentation (presence of CNS features) and the salicylate
concentration in excess of 700 mg/L suggest a severe overdose,
although acidaemia (high [H+]) and renal impairment (also seen in
severe overdose) are not seen here.
(c) Initial treatment options include supportive rehydration (patients are
often dry due to severe sweating), intravenous glucose (CNS glucose
may be much lower than plasma glucose), and potassium. Mortality
from severe salicylate poisoning is ~5% and haemodialysis should be
considered. If this is not available, other options include repeated oral
activated charcoal (shortens plasma half-life of salicylate), and
alkalinization of the urine (increases clearance of salicylate), by giving
intravenous sodium bicarbonate. Acetazolamide also alkalinizes the
urine but also causes a metabolic acidosis and is therefore contra-
indicated. Monitoring in a high-dependency setting is advised.

80
TOXICOLOGY CASE 41

A 56-year-old man is brought to A&E on a Saturday night having


reportedly ingested antifreeze. He is lucid and conscious (Glasgow Coma
Scale 15/15), and reports drinking ‘about half a pint’ of antifreeze
approximately three hours before. He is nauseated and vomits several
times. Although he denies alcohol intake, he occasionally slurs his words;
clinical assessment is otherwise unremarkable. Bloods are taken for
electrolytes (including bicarbonate), glucose and serum osmolality. Results
are shown below.

Na+ 141 mmol/L (135-145)


K+ 3.7 mmol/L (3.4-4.9)
Urea 6.0 mmol/L (2.5-8.0)
HCO3- 22 mmol/L (21-28)
Glucose 5.0 mmol/L (4.0-5.5)
Osmolality 347 mmol/kg (275-295)

(a) Comment on the results.

(b) What else would you request?

(c) What are the treatment options?

81
TOXICOLOGY ANSWER 41

(a) The major biochemical abnormality is the presence of a significant


osmolal gap (the difference between measured serum osmolality and
serum osmolality calculated from electrolytes). One formula used to
calculate the serum osmolality is: 2([Na+] + [K+]) + [urea] +
[glucose]. The factor of two applied to sodium and potassium allows
for associated anions, and assumes complete ionization (there are
minor variations on this formula). Here the above calculation would
give a calculated serum osmolality of 2([141] + [3.7]) + [6] + [5] =
300.4 mmol/kg. Measured serum osmolality was 347 mmol/kg,
giving an osmolal gap of 46.6 mmol/kg (it should be <10
mmol/kg). The size of the osmolal gap here indicates ingestion of a
significant quantity of unmeasured active solute.
(b) The history of antifreeze ingestion and the finding of a significant
osmolal gap suggest that ethylene glycol should be measured.
However, its analysis is complex and most hospital laboratories do
not routinely measure this out of hours and may require persuasion
of the clinical need for urgent measurement. In the context of
reported antifreeze ingestion, the finding of a significant osmolal
gap (indicating unmeasured active solute) or of a metabolic acidosis
(indicating metabolism of ethylene glycol to toxic acidic metabolites
glycolic, glyoxylic and oxalic acids) indicates the need to quantify
ethylene glycol urgently.
(c) The mainstay of treatment is to administer ethanol (alcohol). The
mechanism is that the enzyme alcohol dehydrogenase (which
metabolises ethylene glycol as outlined above) has much greater
affinity for ethanol than for ethylene glycol. This retards the
formation of toxic metabolites. Once blood samples for analysis are
collected, alcohol treatment should be initiated pending the test
results. Severe poisoning requires haemodialysis to eliminate
ethylene glycol, and should be continued until it is undetectable. A
specific antidote is Fomepizole but limited due to its high cost.
(Other measures are supportive: sodium bicarbonate to correct
severe metabolic acidosis; calcium if severely hypocalcaemic;
adequate fluid intake).
82
PAEDIATRICS CASE 42

A 1-year-old, previously well baby boy, was rushed to A&E at 8:30 am on


Monday morning by his parents as they were unable to rouse him to take
him to the nursery. The parents had gone out the previous evening and
therefore had fed the child earlier than usual and put him to bed. (The
babysitter had remarked that the evening had been uneventful). In the
A&E blood samples were sent off to the laboratory urgently. At the
same time, finger prick testing, done by the nurse in A&E, showed a
blood glucose of 3.2 mmol/L. She reported this as ‘normal’. Twenty
minutes later, the laboratory phoned back with a plasma glucose result of
0.9 mmol/L, plasma ammonia of 250 µmol/L and mildly elevated liver
transaminases.

(a) Comment on the blood glucose result.

(b) Establish a differential diagnosis and list the relevant initial


investigations.

(c) What is the likely diagnosis?

(d) In general, what factors in the family, personal and social history are
important pointers towards an inborn error of metabolism?

83
PAEDIATRICS ANSWER 42

(a) The plasma glucose is very low. Handheld glucometers are not
sufficiently accurate at very low or very high glucose concentrations.
If hypoglycaemia is suspected, plasma glucose should always be
confirmed by laboratory testing.
(b) Hypoglycaemia is not uncommon in the first few weeks of life,
especially in premature babies, usually due to poor glycogen reserves,
inadequate feeding or transient hyperinsulinaemia. However, in a
child of this age group, pathological causes should be considered:
poor diet, malabsorption, diarrhoea and vomiting, chronic renal and
liver diseases and infections Once these are ruled out, an Inborn
Error of Metabolism (IEM) is a distinct possibility. This case
highlights a typical history: the child effectively missed a full meal and
was without nutrition for about 14-16 hours; at some point during
the night he became hypoglycaemic and unnoticed, slipped into
hypoglycaemic coma. Initial investigations include full blood count,
plasma glucose, U&E, LFT, arterial blood gases, lactic acid and
ammonia (there should be a low threshold for measuring the lactic
acid and ammonia in seriously ill children). Further samples, for
specialised tests, should be collected (in discussion with the
laboratory) ideally at the time of decompensation, before treatment
(in this case glucose) is initiated. Laboratories often have protocols to
assist with this.
(c) In this case the clinical history, high ammonia and mild liver
dysfunction is a strong pointer towards a fatty acid oxidation (FAO)
defect; this is due to an enzyme defect, which results in an inability to
metabolize fatty acids to ketone bodies (an alternative energy source)
during a prolonged fast; there is an associated accumulation of fatty
acids causing the raised liver enzymes.
(d) Previous sibling death in the neonatal period (where no clear
diagnosis was made), family history of undiagnosed ‘neurological’
illness and consanguinity in parents are some of the factors that
should raise the suspicion of a possible IEM.

84
PAEDIATRICS CASE 43

A 6-day-old term baby girl of consanguineous parents presented at the


Emergency Department with 3-4 episodes of vomiting; she had been well
for the first couple of days of life and then had stopped taking feeds and
become lethargic. Initial blood tests were as follows:

Na+ 142 mmol/L (135-145)


K+ 3.2 mmol/L (3.4-5.3)
Urea 6.7 mmol/L (2.5-5.1)
Creatinine 32 µmol/L (27-62)
Glucose 7.1 mmol/L (4.0-5.5)
Ammonia 270 µmol/L (<100)

Arterial Blood Gases

H+ 31 nmol/L (35-45)
PCO2 3.1 kPa (4.5-5.6)
HCO3- 20 mmol/L (18-26)

(a) Discuss the results in light of the clinical history.

(b) What clinical features should raise suspicion of high ammonia?

(c) What are the causes of high ammonia?

(d) Discuss the principles of management.

85
PAEDIATRICS ANSWER 43

(a) Hyperammonaemia; mild respiratory alkalosis, hypokalaemia and high


urea (due to dehydration).

(b) Clinical features of hyperammonaemia in neonates are non-specific


and include poor feeding, irritability, lethargy, ataxia and convulsions.
Undiagnosed, these may lead to permanent neurological sequelae,
cognitive impairment, mental retardation or death. Older children
may present with poor or declining performance in school or
development of food fads (avoidance of protein); behavioural
changes in adults, may be a rare presentation. Because there are no
pathognomonic clinical features the most important factor is
awareness of hyperammonaemia as a possible causative factor and
testing for it.

(c) The commonest causes of a high measured ammonia are non-


pathological: difficult venepuncture, struggling baby, poor
venepuncture technique, prolonged and tight tourniquet. All of these
can give rise to artefactually high ammonia concentration. Ammonia
samples should be transported to the laboratory (ideally on ice) and
centrifuged within 15-20 minutes, else continued cellular metabolism
can cause an in-vitro rise. Pathological causes to be included in the
differential diagnosis are renal disease, liver dysfunction and drugs
e.g. anti-epileptics. However, high ammonia should raise suspicion of
IEMs: urea cycle disorders, organic acidurias, fatty acid oxidation
defects and respiratory chain disorders.

(d) Once hyperammonaemia is detected, repeated and frequent ammonia


measurements are required as the ammonia concentration may rise
very quickly, and cause rapid and permanent neurological damage.
The aims of treatment are to remove ammonia from the body quickly
and to minimise further production. Protein intake is stopped and
intravenous 5% dextrose infused. Sodium benzoate or sodium
phenylbutyrate is used to ‘scavenge’ the ammonia. Very high
ammonia concentrations require haemodialysis in specialist paediatric
centres with early involvement of a metabolic specialist.
86
PAEDIATRICS CASE 44

A 12-day-old baby boy was admitted to hospital because of increasing


jaundice. Clinically, he was thriving and well and taking regular breast
feeds. Examination was unremarkable other than the obvious jaundice.
His blood test results were as follows:

Na+ 139 mmol/L (135-145)


K+ 4.6 mmol/L (3.5-5.3)
Urea 2.5 mmol/L (1.0-5.0)
Creatinine 40 µmol/L (27-62)

Tot. Bilirubin 220 µmol/L (0-50)


Conj. Bilirubin 8 µmol/L (<10)
ALT 35 U/L (5-60)
AST 58 U/L (40-120)
Albumin 29 g/L (25-35)
Tot Protein 56 g/L (50-70)

CRP < 3mg/L (< 3)

(a) What is the likely cause of the baby’s jaundice?

(b) When should neonatal jaundice be further investigated?

87
PAEDIATRICS ANSWER 44

(a) The baby is clinically well and taking proper feeds; other than the
high serum bilirubin, most of which is unconjugated, the blood test
results are within reference limits. In this case the most likely cause is
breast-milk jaundice. This condition may be due to: i) the presence of
a factor in the breast milk that reduces the action of the liver enzyme
responsible for the conjugation of bilirubin. (Conjugated bilirubin is
water soluble and excreted in the urine, whereas unconjugated
bilirubin accumulates in the blood and stains the skin, sclera and
various other tissues yellow.) or ii) in the adult gut the microbial flora
are responsible for facilitating the faecal excretion of conjugated
bilirubin; in the newborn, the absence of this mechanism results in
the deconjugation of the already conjugated bilirubin by enzymes
present in the gut wall and reabsorption of this unconjugated
bilirubin into the blood stream. The diagnosis of breast milk jaundice
should be made only after a detailed clinical assessment to rule out
any pathological causes of jaundice.

(b) Physiological jaundice (due to immaturity of the liver conjugation


enzymes and increased bilirubin production because of breakdown of
fetal red cells) is seen in nearly half of term healthy babies and it starts
after the first 24 hours of life. The bilirubin moiety is almost entirely
of the unconjugated variety. If jaundice starts within 24 hours of birth
then it is almost always pathological.

In a term baby, physiological jaundice does not persist beyond 2


weeks; if it does then it is defined as prolonged jaundice. Prolonged
jaundice may be due to a variety of clinical conditions including
breast milk jaundice, continuing haemolysis, bruising, sepsis, systemic
illness, metabolic disorder and malformation of the hepatobiliary
system. In many (not all) of these conditions an important marker is
the presence of increased amounts of conjugated bilirubin (normally
less than 10% of total bilirubin). Therefore, any child presenting with
very early onset or prolonged jaundice requires detailed investigations
to elicit the cause, as does the presence of increased conjugated
bilirubin or concomitant illness.

88
MISCELLANEOUS CASE 45

A 46-year-old woman was started on lithium therapy for bipolar disorder.


After a couple of months she noticed increased urinary frequency and
nocturia with increased thirst and lack of energy. The GP sent off blood
and urine samples the results of which are shown below:

Na+ 146 mmol/L (135-145)


K+ 4.9 mmol/L (3.4-4.9)
Urea 9.1 mmol/L (2.5-8.0)
Creatinine 95 µmol/L (40-130)
Glucose 5.1 mmol/L (4.0-5.5)

Bilirubin 12 µmol/L (3-22)


ALT 44 U/L (3-55)
Alk Phos 110 U/L (80-280)
Adj Calcium 2.39 mmol/L (2.20-2.60)

Serum osmolality 306 mmol/kg (280-295)


Urine osmolality 80 mmol/kg
Urine sodium 28 mmol/L

(a) Comment on the biochemistry

(b) What is the likely cause of her symptoms?

89
MISCELLANEOUS ANSWER 45

(a) Diabetes mellitus and hypercalcaemia (as potential causes of the


patient’s symptoms) are effectively ruled out by the normal glucose
and adjusted calcium concentrations, respectively. High serum
sodium and high urea are consistent with dehydration due to the
increased urinary fluid loss. The extracellular fluid depletion also gives
rise to weakness and lack of energy. The high serum osmolality is in
keeping with the high urea and sodium. It may also be calculated as
detailed in Case 41. It is noticeable that there are no reference
intervals provided for urine osmolality and urine sodium
concentration. These vary considerably depending on the state of
hydration, medications and diet. Maximally dilute urine has an
osmolality of <100 mmol/kg.

(b) The most likely diagnosis is lithium-induced nephrogenic diabetes


insipidus. This is a known side-effect of chronic lithium therapy but
onset of symptoms maybe within a few weeks of starting treatment.
Lithium inhibits the action of antidiuretic hormone on the distal renal
tubule, impairing water reabsorption. The diagnosis may be
confirmed by performing a water deprivation test, which is not
without potential risks and should be performed under specialist
supervision.

Treatment is by discontinuation of lithium therapy and then


reintroduction at a lower dose. In some cases symptoms may persist
long after discontinuing lithium. Other renal manifestations of
chronic lithium treatment are interstitial nephritis and renal tubular
acidosis. The most important endocrine disorder secondary to long-
term use is hypothyroidism. Lithium is taken up by thyroid cells and
blocks the action of TSH, thyroid hormone synthesis and the release
of preformed thyroid hormones. Patients on chronic lithium therapy
undergo regular monitoring of their serum lithium concentration as it
has a very narrow therapeutic index, i.e. the therapeutic and toxic
concentrations are very close.

90
MISCELLANEOUS CASE 46

A 65-year-old retired man made an appointment with his GP to get his


PSA test done. He was in good health, but most members of his bowling
team had been tested, and he also wanted to rule out the possibility of
prostate cancer. The GP carried out a per rectal examination and requested
routine blood tests and a PSA.

Na+ 141 mmol/L (135-145)


K+ 3.9 mmol/L (3.4-4.9)
Urea 4.5 mmol/L (2.5-8.0)
Creatinine 98 µmol/L (40-130)

Bilirubin 14 µmol/L (3-22)


ALT 38 U/L (3-55)
Alk Phos 110 U/L (80-280)
Albumin 43 g/L (40-52)
Total protein 77 g/L (60-80)

PSA 7.9 µg/L (0-4)

(a) Comment on the PSA result.

(b) Discuss the pros and cons of PSA as a population screening test for
the diagnosis of prostate cancer.

91
MISCELLANEOUS ANSWER 46

(a) The PSA is modestly elevated to about twice the upper reference
limit. Although this may be consistent with early prostate cancer, it
can also be high due to a variety of other reasons including benign
prostatic hypertrophy, prostatitis, urinary tract infections, urinary
obstruction or following a per rectal examination.

(b) Population screening for prostate cancer using PSA presents a


challenging dilemma. It does fulfil some of the requirements of a
screening marker: availability of a cheap and simple blood test to
detect high PSA; early detection of prostate cancer in the
asymptomatic stage, which is potentially curable; and well-established
modalities of treatment. On the other hand one of the biggest
drawbacks of the test is its lack of sensitivity and specificity. Many
cancers are not associated with a PSA increase (false negative test),
especially in the early stages, which may lead to cancer being missed.
As mentioned above, PSA increases non-specifically in many
conditions other than prostate cancer. This may lead to unnecessary,
invasive, stressful and expensive investigations. Not all prostate
cancers require treatment and the knowledge of the presence of even
a slow-growing cancer in one’s body may give rise to anxiety and
reduce the quality of life. Treatment of prostate cancer may be
associated with significant side-effects such as urinary incontinence
and erectile and bowel dysfunction. It is as yet not clear whether PSA
testing has directly contributed to a reduction in deaths from prostate
cancer.

92
MISCELLANEOUS CASE 47

A 26-year-old man consulted his GP complaining of fatigue and lack of


energy. On detailed questioning it emerged that he had had similar
symptoms for a couple of years, but had kept ignoring them as he worked
a mix of day and night shifts and attributed his symptoms to his altered
sleep pattern. There was no significant past medical history; no history of
recent travel, alcohol or drug abuse. The GP requested some blood tests.

Hb 17.1 g/L (11.5-18)


Total and differential white cell count were normal

Na+ 141 mmol/L (135-145)


K+ 3.9 mmol/L (3.4-4.9)
Urea 4.5 mmol/L (2.5-8.0)
Creatinine 98 µmol/L (40-130)

Bilirubin 14 µmol/L (3-22)


ALT 175 U/L (3-55)
Alk Phos 110 U/L (80-280)
TSH 2.3 mU/L (0.4-4.0)
fT4 16 pmol/L (9-22)

Iron 36 µmol/L (5-32)


Transferrin 1.5 g/L (2.1-4)
Ferritin 670 µg/L (30-240)
Transferrin Sat 96% (20-55)

(a) What diagnosis is strongly suggested by the results?

(b) How may the diagnosis be confirmed?

(c) If left untreated, what is the usual course of the disease?

93
MISCELLANEOUS ANSWER 47

(a) In a young man, chronic fatigue is an unusual symptom. Anaemia,


thyroid disease and depression are important differential diagnoses; if
these have been excluded, rarer causes like hereditary
haemochromatosis should also be considered. In this case, Hb and
thyroid function tests rule out anaemia and thyroid disease as a possible
cause of his symptoms. Elevated transferrin saturation (>50% in a
female or > 55% in a male) along with increased ferritin is consistent
with iron overload. These results are strongly suggestive of hereditary
haemochromatosis. (Other causes of iron overload include excess
alcohol, multiple blood transfusions, chronic haemolysis and excess
parenteral iron therapy). It is usual practice to repeat iron studies in a
morning (fasting) blood sample. This is more important when
haemochromatosis is clinically suspected, but the test results do not
confirm it. (There is diurnal variation with peak serum iron
concentrations in the morning dropping significantly in the afternoon).
The basic pathology in hereditary haemochromatosis is unregulated
iron absorption leading to iron accumulation. The elevated ALT is
supportive of this diagnosis and suggestive of hepatic parenchymal
damage. Hepatic dysfunction (manifested as elevated liver enzymes)
often brings the diagnosis to light.
(b) The diagnosis is confirmed by genetic testing for the common
mutations in the HFE gene: C282Y (90% of the cases) and H63D. In
the Caucasian population, hereditary haemochromatosis has a genetic
prevalence of approximately 1 in 150, although a much smaller
proportion of those affected actually develop the disease. It is
important to counsel the patient for a positive family history.
(c) Untreated, iron accumulation may affect various organs: hepatic
parenchymal damage, ultimately leading to cirrhosis and liver cancer;
exocrine pancreatic insufficiency and diabetes (so called ‘bronze
diabetes’ due to associated grayish pigmentation of the skin); cardiac
arrhythmias and congestive cardiac failure; arthritis; hypogonadism; and
thyroid dysfunction. Early diagnosis and treatment (venesection and
blood letting) limits the progression of disease and minimises end
organ damage.

94
MISCELLANEOUS CASE 48

A 57-year-old woman was admitted with vague abdominal pain and


nausea. BP was 145/98; pulse 88/min regular, respiratory rate 14/minute;
auscultation of the heart and chest was unremarkable. The junior doctor
requested the following blood tests:
Na+ 142 mmol/L (135-145)
K+ 4.2 mmol/L (3.4-4.9)
Urea 5.1 mmol/L (2.5-8.0)
Creatinine 67 µmol/L (40-130)
Adj Calcium 2.33 mmol/L (2.20-2.60)
Phosphate 0.79 mmol/L (0.7-1.4)
Amylase 1578 U/L (0-100)
CRP <5 mg/L (<3)
Bilirubin 12 µmol/L (3-22)
ALT 24 U/L (3-55)
Alk Phos 112 U/L (80-280)
Total protein 78 g/L (50-70)
Albumin 41 g/L (40-52)

(a) Comment on the blood test results.


(b) Discuss the differential diagnosis of a raised serum amylase.

The junior doctor made a provisional diagnosis of acute pancreatitis and


asked the surgical registrar to review the patient. At the time of the surgical
review the patient was asymptomatic and repeat serum amylase was 1800
U/L; the surgeon confirmed the diagnosis. Over the ensuing months the
serum amylase concentrations were monitored and remained high; the
patient remained asymptomatic. Despite normal abdominal imaging, she
was diagnostically labelled as having sub-acute, chronic or relapsing
pancreatitis throughout this course.
(c) What is macroamylase?
(d) What are the causes of asymptomatic hyperamylasaemia?

95
MISCELLANEOUS ANSWER 48

(a) Very high serum amylase (disproportionate to the patient’s symptoms);


other biochemistry unremarkable.
(b) If patients present with acute abdominal pain a raised (usually more
than 10 times the upper reference limit) serum amylase is almost always
diagnostic of acute pancreatitis. Other abdominal causes of high
amylase include ruptured tubal pregnancy, perforated hollow
abdominal viscus, small bowel obstruction and generalised peritonitis.
Salivary glands have high amylase content leading to increased serum
amylase when they are affected by various pathologies e.g.
inflammation, calculi or malignancy. Hyperamylasaemia is also noted in
diabetic ketoacidosis (actual mechanism not clearly elucidated), chronic
renal failure and following administration of opiates (by causing spasm
of sphincter of Oddi).
(c) Sometimes amylase molecules combine with plasma proteins (usually
IgG). The increased molecular mass (macro) of the resultant complexes
is large enough to preclude renal excretion, unlike amylase which has a
relatively low mass and is easily filtered by the kidney.
Macroamylasaemia is essentially a benign phenomenon as the
complexes are not biologically active. Macroamylase (and various other
macroenzymes and macrohormones) are well recognised by laboratory
staff and if suspected can be readily confirmed or excluded. It is
important to note that a diagnosis of macroamylase should not be
made primarily until other pathological causes have been excluded.
(d) Macroamylase has to be excluded in all causes of asymptomatic
hyperamylasaemia. Sometimes pancreatic pseudocysts (circumscribed
collections of fluid around the pancreas) may form during or after
episodes of acute pancreatic necrosis. The pseudocysts may retain high
amylase activity, which leaks into the blood, and may persist even after
the patient has become asymptomatic. Another very rare but
important cause is malignant hyperamylasaemia and various tumours
have been documented to produce amylase. This patient was ultimately
diagnosed as having macroamylase (there was no pancreatic pathology).
This case also highlights the importance of correlating clinical and
laboratory data.
96
MISCELLANEOUS CASE 49

A 29-year-old woman presented to her GP complaining of weakness and


inability to concentrate. A few years ago, she had been diagnosed with
irritable bowel syndrome, and intermittently suffered from dyspepsia. Her
body mass index was 19 kg/m2 and had been so since her teens. Routine
blood test results were as follows:

Na+ 135 mmol/L (135-145)


K+ 3.6 mmol/L (3.4-4.9)
Urea 1.8 mmol/L (2.5-8.0)
Creatinine 44 µmol/L (40-130)
Bilirubin 13 µmol/L (3-22)
ALT 33 U/L (3-55)
Alk Phos 32 U/L (80-280)
Albumin 27 g/L (40-52)

Adj Calcium 1.98 mmol/L (2.20-2.60)


Phosphate 0.59 mmol/L (0.7-1.4)

Total protein 56 g/L (50-70)


Globulins 29 g/L (17-35)

TSH 2.4 mU/L (0.4-4)


fT4 14 pmol/L (9-22)

Hb 9 g/L (11.5-18)
Peripheral smear: microcytosis and hypochromia

(a) Comment on the results.

(b) What is the most likely diagnosis?

(c) What tests are usually done to confirm the diagnosis?

97
MISCELLANEOUS ANSWER 49

(a) Hypochromic microcytic anaemia along with hypocalcaemia and


hypophosphataemia are the striking abnormalities. Also note that
urea and creatinine are low, most probably reflecting her slight build.

(b) Blood test results point towards a malabsorptive pathology; Coeliac


disease is the most likely diagnosis. This results in malabsorption of
many nutrients, including minerals and vitamins. Chronic, often non-
specific, gastro intestinal symptoms, unexplained anaemia, weight loss
(or inability to put on weight) and clinical features suggestive of
vitamin deficiencies are pointers towards the correct diagnosis.
Rarely, patients are asymptomatic and discovered incidentally when
routine tests are carried out.

(c) Coeliac disease is an autoimmune disorder of the small intestine


caused by hypersensitivity to gluten protein found in wheat and
similar grains. This results in inflammation of the gut wall, leading to
atrophy of the intestinal villi, which results in a reduced surface area
for absorption. Diagnosis is usually made by finding increased blood
concentrations of antibodies to the intestinal enzyme tissue
transglutaminase. In some patients endoscopy and duodenal biopsy
may be necessary to establish a histological diagnosis. Treatment is
with a life-long gluten-free diet.

98
MISCELLANEOUS CASE 50

A 52-year-old woman known to have a history of chronic alcohol abuse,


was admitted to hospital with jaundice, abdominal pain and distension.
The on-call registrar made a clinical diagnosis of alcoholic cirrhosis with
ascites and requested an ultra sound of the abdomen and blood tests
including CA-125, as he felt that ovarian malignancy was an important
differential diagnosis.

The blood test results confirmed hepatic dysfunction. CA-125 was


reported as 86 kU/L (premenopausal <35; post menopausal <25).

How would you interpret the CA-125 result in this case?

99
MISCELLANEOUS ANSWER 50

CA-125 is a tumour marker which is used to monitor treated patients of


ovarian malignancy to assess their response to treatment as well as to
detect recurrence. However, it is not specific for ovarian cancer. It may
also be high in other cancers, e.g. those of the breast, gut, lung and uterus.
What adds to the diagnostic conundrum is that menstruation, pregnancy,
endometriosis and pelvic inflammatory disease may also be associated with
high CA-125.

CA-125 is normally found in cells of mesothelial origin (such as


pericardium, pleura and peritoneum) as well as some of non-mesothelial
origin (cervical and bronchial epithelium). Thus, inflammation of any of
these tissues may result in increased CA-125 secretion. This property
accounts for the non-specificity of CA-125. It is also not sensitive enough
to definitively detect early ovarian cancers; in fact it may not be elevated in
about 20% of patients with confirmed ovarian malignancy. This limits its
use as a screening tool for ovarian cancer except under specific
circumstances.

In this patient, the high CA-125 is most probably due to the ascites,
resulting from cirrhosis of the liver.

100
MISCELLANEOUS CASE 51

A 78-year-old woman who had been suffering from lower back pain for
several years, went to her GP as she had experienced a significant increase
in the intensity of the pain over the last couple of months. There was no
history of recent trauma. The GP increased her pain medications and
referred her for a lumbar spine x-ray and routine blood tests. The results
were as follows:

Hb 10.4 g/L (11.5-18.0)


WBC 8 x 109/L (4-11 x 109)
ESR 26 mm/hour (0-15)
Na+ 137 mmol/L (135-145)
K+ 4.8 mmol/L (3.4-4.9)
Urea 8.9 mmol/L (2.5-8.0)
Creatinine 145 µmol/L (40-130)

Adj Calcium 2.79 mmol/L (2.20-2.60)


Phosphate 1.33 mmol/L (0.7-1.4)
Alk Phos 130 U/L (80-280)

Bilirubin 18 µmol/L (3-22)


ALT 48 U/L (3-55)
Alk Phos 980 U/L (80-280)
Albumin 38 g/L (40-52)
Total protein 92 g/L (50-70)
Globulins 54 g/L (17-35)

X-ray lumbar spine was reported as: Widespread degenerative changes present in
the lumbar spine. There is a grade 2 wedge compression fracture of L2. The bones in
general appear osteopenic.

(a) In light of the clinical history discuss the blood test results and the
significance of the x-ray findings.
(b) What is the most likely diagnosis?
(c) What further investigations should be carried out to confirm the
diagnosis?
101
MISCELLANEOUS ANSWER 51

(a) The significant abnormalities are anaemia, hypercalcaemia, impaired


renal function and elevated total globulins (resulting in a high total
protein concentration). In this age-group high calcium should raise the
suspicion of malignancy (the other common cause of hypercalcaemia
being primary hyperparathyroidism). The widespread degenerative
changes in the lumbar spine are not unusual for age; the lumbar
vertebra compression fracture is significant and should raise the
possibility of osteoporosis or malignancy. Increased globulins and ESR
are non specific markers of inflammation.

(b) Collectively, the patient’s age, hypercalcaemia, increased total globulins,


vertebral compression fracture and renal impairment strongly suggest a
diagnosis of multiple myeloma. The differential diagnosis of skeletal
metastasis should also be kept in mind. Cancers that commonly
metastasize to bone include breast, lung, thyroid, kidney and (in men)
prostate. Multiple myeloma is a cancer of plasma cells, which are found
in the bone marrow, and are antibody-producing modified B
lymphocytes. It is rare before the age of 40 and its incidence increases
with rising age. It is often asymptomatic, and therefore diagnosis may
be delayed for many years. Anaemia (due to suppression of the bone
marrow by the cancer cells), renal impairment, hypercalcaemia and
fracture (especially vertebral compression) are important signs of more
advanced stages of myeloma.

(c) The basic pathology in multiple myeloma is malignant transformation


of plasma cells, which proliferate and invade the bone marrow. A bone
marrow biopsy is required to estimate the percentage of bone marrow
occupied by the malignant plasma cells. They produce abnormal
immunoglobulins, which accumulate in the blood and are also excreted
in the urine (Bence-Jones proteins). These can be identified by serum
and urine protein electrophoresis as a paraprotein (M-band). X-rays of
the pelvis and spine often show lytic lesions consistent with skeletal
spread.

102
MISCELLANEOUS CASE 52

A 45-year-old man, who had been previously fit and well, was admitted to
ITU after a hit-and-run accident resulting in multiple fractures and
subdural haematoma. He underwent craniotomy and decompression. Over
the next few days he remained haemodynamically stable, but deeply
comatose and required continuous ventilation; he was fed enterally via a
nasogastric tube. There was no history of alcohol or recreational drug
abuse and he had no significant past medical history. After five days an
anaesthetist trainee decided to request a micronutrient screen along with
routine bloods.

The results, which were available a few days later, were as follows:

Zinc 8 mmol/L (12-18)


Selenium 0.3 mmol/L (0.8-2.0)
Vit A 0.5 mmol/L (1.0-2.8)
Vit E 10 mmol/L (15-40)
Vit C 10 µmol/L (15-90)
Vit B6 13 nmol/L (20-140)
Beta-Carotene 34 µg/L (90-310)

Other relevant results were:

Total protein 45 g/L (60-80)


Albumin 16 g/L (35-50)
Globulin 29 g/L (17-35)
CRP 120 mg/L (<3)

(a) Comment on the results of the micronutrient screen.

(b) What is the significance of the low albumin and high CRP?

(c) Is there an indication for supplementing trace elements?

103
MISCELLANEOUS ANSWER 52

(a) There is a significant decrease in the plasma concentration of all


measured vitamins and trace elements, which, at initial perusal, may
indicate that the patient has developed a deficiency of micronutrients.

(b) CRP is a positive acute-phase reactant and rises with acute


inflammation and/or infection. At the same time, serum albumin is a
negative acute-phase reactant and most patients in the ITU have a
very low concentration. These confirm the presence of on-going
systemic inflammation and have important implications for
interpreting the results of micronutrient analysis.

(c) Most trace elements and micronutrients are negative acute-phase


reactants. When measured in the presence of acute inflammation the
results will be artefactually low, which may be interpreted as
micronutrient deficiency. This may be explained as follows: in the
acutely inflamed state there is a generalized increase in capillary
permeability resulting in the leakage of albumin into the extracellular
space. Many of the micronutrients are bound to albumin, thus
causing a transient fall in their measured concentration in plasma.
Once inflammation subsides, albumin and micronutrient
concentrations return to normal.

Other possible causes of a fall in micronutrient concentration in the


presence of acute inflammation include their sequestration in the liver
and other organs, increased utilisation (in the catabolic state) and
increased renal excretion.

The short acute illness of a few days here is not usually sufficient to
result in micronutrient deficiency, especially as there was no clinical
history to suggest that prior to the accident the patient was
nutritionally compromised. Therefore, supplementation is not
indicated at present.

104
MISCELLANEOUS CASE 53

A 58-year-old man, while travelling, experienced chest pain radiating to the


left arm accompanied with sweating. The pain passed within a few hours
and he did not seek medical advice. He returned home after a couple of
days and made an appointment to see his GP, who carried out an ECG
and requested Troponin-T (TnT) along with routine blood tests. The
ECG showed non-specific ST wave changes. The blood test results were
as follows:

Na+ 144 mmol/L (135-145)


K+ 4.6 mmol/L (3.4-4.9)
Urea 7.8 mmol/L (2.5-8.0)
Creatinine 108 µmol/L (40-130)
Bilirubin 19 µmol/L (3-22)
ALT 100 U/L (3-55)
Alk Phos 200 U/L (80-280)
Albumin 38 g/L (40-52)

Troponin – T 0.78 µg/L (<0.01)

a) Comment on the TnT result.

b) Explain the high ALT.

105
MISCELLANEOUS ANSWER 53

a) TnT is elevated which, along with the clinical history, is consistent


with the patient having had an acute myocardial infarction (MI).
The term acute MI should be used when there is evidence of
myocardial necrosis in a clinical setting consistent with myocardial
ischaemia. Under these conditions one of the criteria that meets
the diagnosis of MI is: detection of rise and/or fall of Troponin
with at least one value above the 99th percentile of the upper
reference limit with at least one of the following:

• Symptoms of ischaemia
• ECG changes of new ischaemia (new ST-T changes or
new left bundle branch block (LBBB))
• Development of pathological Q waves in the ECG
• Imaging evidence of new loss of viable myocardium or
new regional wall motion abnormality

How long does TnT remain elevated after an acute event? It starts
rising within a few hours after myocardial cell necrosis and peaks
around 48 hours; it remains detectable for 10-12 days. This makes
it a valuable marker for the diagnosis of MI in patients presenting
late. This patient presented at least 2 days after an acute event and
as expected TnT was still elevated.

b) ALT is not specific to the liver (as is a common misconception).


It is present in a variety of tissues including cardiac muscle. The
ALT rise is consistent with release from myocardial cell necrosis.
ALT would also be elevated if the patient were to develop acute
congestive cardiac failure, but the clinical presentation would
include respiratory distress, hypotension, peripheral oedema and
tachycardia.

106
MISCELLANEOUS CASE 54

A 22-year-old woman was brought to the A&E with complaints of severe


abdominal pain and vomiting. (Clinical history revealed that this was her
third admission in the last 18 months, with similar symptoms, but no
diagnosis had been made. Previous routine blood tests and abdominal
ultrasound had been normal and her symptoms had settled with pain relief
and supportive management). On examination, there was diffuse
abdominal tenderness but no rigidity; systemic examination was
unremarkable. Blood was collected for routine tests, as well as urine for
dipstick testing. The nurse doing the dipstick testing noticed that it was
very dark coloured and was surprised when it turned out to be negative for
protein, blood, nitrites and bilirubin. Blood test results were as follows:

Na+ 131 mmol/L (135-145)


K+ 4.2 mmol/L (3.4-4.9)
Urea 3.8 mmol/L (2.5-8.0)
Creatinine 89 µmol/L (40-130)

Adj Calcium 2.35 mmol/L (2.20-2.60)


Phosphate 0.90 mmol/L (0.7-1.4)
Albumin 40 g/L (40-52)
Alk Phos 110 U/L (80-280)
ALT 33 U/L (3-55)
Bilirubin 8 µmol/L (3-22)

Amylase 68 U/L (0-100)


HCG <3 U/L (<3)

a) Comment on the blood results and what important diagnosis


should be considered in this case?

b) How may this diagnosis be confirmed?

107
MISCELLANEOUS ANSWER 54

(a) There is a mild hyponatraemia and the serum amylase is not raised
(effectively ruling out acute pancreatitis); pregnancy test is negative.
When the usual causes of acute abdominal pain e.g. acute
appendicitis, ruptured hollow viscus, cholecystitis, acute hepatitis and
(in a woman of child-bearing age) ruptured ectopic pregnancy, have
been ruled out, acute porphyria should be considered in the
differential diagnosis. Porphyrias are a group of heterogenous genetic
disorders due to enzyme deficiencies in the Haem synthesis pathway.
Though individually rare, their combined incidence in the UK is
approximately 1 in 20,000. The commonest acute porphyria in the
UK is Acute Intermittent Porphyria (AIP). AIP usually presents after
puberty with acute abdominal pain, behavioural or neuropsychiatric
symptoms. Common precipitating factors include hormonal changes
due to pregnancy or menstruation, excess alcohol, fasting, infections
and a variety of medications including antiepileptics. Treatment is
supportive: elimination of precipitating factor, pain control (usually
with opiates) and intravenous dextrose. Specific treatment is with
haem arginate, which reduces accumulation of toxic metabolites.
(b) Initial diagnosis is established by testing for porphobilinogen in the
urine. This accumulates in the body due to the enzyme deficiency and
is excreted in large amounts. The dark colour of the urine is due to
spontaneous polymerization of porphobilonogen to porphyrin (Greek:
porphyra = purple pigment). Acute porphyria can be ruled out if
urine porphobilinogen is not elevated when the patient is
symptomatic (it tends to return to normal in between attacks). The
urine sample should be protected from light to avoid a false negative
test. (Mild hyponatraemia, as seen in this case, (due to increased
ADH secretion), can be a tell-tale sign). Further characterization of
the type of porphyria requires specialized testing, done in a reference
laboratory. Genetic testing is useful for family studies. It is important
to be aware of acute porphyria as a possible cause of undiagnosed
acute abdominal pain and, if suspected, have a low threshold for
discussion with the laboratory. If not included in the differential
diagnosis, patients may end up undergoing unnecessary laparotomy.

108
APPENDIX – ACID BASE STRATEGY

Always approach acid-base problems in a systematic way.

• First thing to look at is H+ concentration (pH if those are the


units you use). If it’s raised, then you’re dealing with an acidosis; if
reduced, an alkalosis.

• It does not matter in principle whether you look at the PCO2 or


the bicarbonate next – both are components of the ratio that
gives rise to the H+ concentration. It makes sense to see if the
history points to one or the other. Indeed, it is difficult to
interpret blood gases meaningfully in the absence of a history.
Some people look at the more abnormal of PCO2 and
bicarbonate. This too is a logical approach – whatever is more
abnormal is likely to be the primary problem.

• Whichever one of PCO2 or bicarbonate you have looked a first –


now look at the other one. Check to see if it’s altered in the same
direction (indicating likely compensation) or in a different
direction (indicating a possible mixed acid-base disturbance).

• Always relate to history. This is not a trivial point. Many (perhaps


most) sets of blood gas results can be interpreted in more than
one way – the only way to ‘make sense’ of them is to interpret in
the light of the clinical details; after all, the purpose in doing blood
gases is to shed light on what is wrong with your patient. And, an
arterial stab should not be lightly undertaken.

109
SUGGESTED FURTHER READING

GENERAL

Gaw A, Murphy MJ, Cowan RA, O’Reilly DStJ, Stewart MJ & Shepherd
J. (2008) Clinical Biochemistry: An Illustrated Colour Text 4th Edition,
Churchill Livingstone Elsevier, Edinburgh.

ISBN-13: 978-0443069321
The cases in this book are built around the topics covered in our textbook
Clinical Biochemistry: An Illustrated Colour Text, which is currently in its fourth
edition. If you wish to read further about any of the topics covered in the
cases you will find more detailed and fully illustrated sections in this book.

110
SUGGESTED FURTHER READING

Should you wish to delve even more deeply into any of the topics we have
covered there are a number of other more specialised options. There are,
of course, several massive and prohibitively expensive tomes that cover
every esoteric detail of each topic. We have tended to ignore these as the
average student will have neither the time, the inclination nor the money
to invest in such works. Instead, we have focussed on shorter, but highly
accessible specialist texts, which you are likely to find in most medical
libraries and which, if you choose to buy them, will not break the bank.

FLUID AND ELECTROLYTES

Scott W. (2010) Fluids and Electrolytes Made Incredibly Easy!


Lippincott Williams & Wilkins, London.
ISBN-13: 978-1901831153

This highly accessible textbook is aimed primarily at nursing students, but


would be useful for anyone coming to the topic of fluid and electrolytes
for the first time.

ACID-BASE & RENAL

O’Callaghan C. (2009) The Renal System at a Glance. 3rd Ed. Wiley-


Blackwell, Chichester.
ISBN-13: 978-1405184724

Fully illustrated in colour, this book provides good overviews of renal


physiology and pathology. It includes sections on acid-base balance and
calcium metabolism.

Goldsmith D, Jayawardene S & Ackland P (Eds) (2007) ABC of Kidney


Disease. BMJ Books, Wiley-Blackwell, London.
ISBN-13: 978-1405136754

111
SUGGESTED FURTHER READING

A short (82 A4 pages) overview of renal disease that is illustrated in colour


throughout and is aimed at GPs and junior doctors.

Ward JT, Ward J & Leach RM. (2010) The Respiratory System at a
Glance 3rd Ed. Wiley-Blackwell, Chichester.
ISBN-13: 978-1405199193

This book contains good sections on oxygen transport, acid-base balance


and ventilation-perfusion mismatching, as well as the wide spectrum of
respiratory pathologies. Illustrated in full colour throughout.

LIVER FUNCTION

Mahl T & O’Grady J. (2006) Fast Facts: Liver Disorders. Health Press
Ltd, Oxford.
ISBN-13: 978-1903734735

A small, but useful, book that outlines the key liver pathologies and
includes sections on the interpretation of liver function tests.

Sargent S. (ed) (2009) Liver Diseases: An Essential Guide for Nurses


and Health Care Professionals. Wiley-Blackwell, Chichester.
ISBN-13: 978-1405163064

This book presents detailed sections on a wide range of liver diseases


including cirrhosis, portal hypertension, alcoholic liver disease, viral
hepatitis, autoimmune hepatitis, Wilson's disease and acute liver failure.

112
SUGGESTED FURTHER READING

ENDOCRINE & DIABETES

Hinson JP, Raven P & Chew SL. (2010) The Endocrine System: Systems
of the Body Series. Churchill Livingstone, Edinburgh.
ISBN-13: 978-0702033728

The anatomy, physiology and biochemistry of the endocrine system are


explained using a fully illustrated, integrated approach.

Chew SL & Leslie RG. (2005) Clinical Endocrinology and Diabetes: An


Illustrated Colour Text. Churchill Livingstone, Edinburgh.
ISBN-13: 978-0443073038

Another in the Illustrated Colour Text series. This, like the others, is fully
illustrated in colour throughout and covers diabetes as well as the range of
other endocrine conditions.

Holt T & Kumar S (eds). (2010) ABC of Diabetes. 6th Edition. BMJ
Books, Wiley-Blackwell, Chichester.
ISBN-13: 978-1405177849

A collection of articles from the BMJ that present an excellent overview of


the aetiology, diagnosis and management of Types 1 and 2 diabetes, their
detection and prevention, and the organization of care and support.

REPRODUCTION
Heffner L & Schust DJ. (2010) The Reproductive System at a Glance.
3rd Ed. Wiley-Blackwell, Chichester.
ISBN-13: 978-1405194525

A guide to the physiology, pathology and clinical biochemistry of the


113
SUGGESTED FURTHER READING

human reproductive system. This book is fully illustrated in colour


throughout and contains sections on the investigation of infertility.

POISONING

Timbrell J. (2001) Introduction to Toxicology, 3rd Ed. Taylor &


Francis, London.
ISBN-13: 978-0415247634

There are many much larger toxicology textbooks, but this one at just over
200 pages covers the background and clinical importance of most of the
different forms of poisoning you are likely to encounter. Available as a
paperback and, much less expensively, as a Kindle download.

LIPIDS AND LIPOPROTEINS

Nicholls P & Young I. (2009) Lipid Disorders. OUP, Oxford.


ISBN-13: 978-0199569656

This is a short pocketbook of just over 100 pages, but packed with clear
summaries of lipid and lipoprotein metabolism and detailed explanations
of lipid lowering strategies in clinical practice.

ANALYSIS AND MEASUREMENT IN CLINICAL BIOCHEMISTRY

In this book we have not considered how we measure different analytes


such as sodium, glucose or thyroxine, in practice, but if you wish to learn
more about these topics there are a range of excellent reference works you
may consult.

114
SUGGESTED FURTHER READING

Glencross H, Ahmed N & Wang Q. (Eds) (2010) Biomedical Science


Practice: experimental and professional skills (Fundamentals of
Biomedical Science). OUP, Oxford.
ISBN-13: 978-0199533299

At over 600 pages this multi-authored textbook presents a wealth of


information – all well-written and beautifully illustrated in colour –
covering the background to all the analytical techniques that are used in
clinical laboratories together with a range of useful chapters specifically
written for those wishing to pursue a career in Biomedical Science.

Burtis CA, Ashwood ER & Bruns DE (Eds) (2007) Tietz Fundamentals


of Clinical Chemistry, Saunders, Philadelphia.
ISBN-13: 978-0721638652

At nearly 1,000 pages this is described as the “condensed, student-friendly


version of Tietz Textbook of Clinical Chemistry”. Although we said we
would leave out the giant tomes of our subject from this list of suggested
reading, we felt the need to include at least one, so that you would know
where to find the definitive answer to any question related to clinical
biochemistry. This book should be in every medical library and, generally
speaking, if it is not in “Tietz” it is not worth knowing.

115
USEFUL WEBSITES

www.aacc.org/publications/clin_chem/casestudies/

If you are looking for longer, more in-depth and altogether more
challenging case studies in this topic then you should look at the on-line
archive of cases published by the American Association of Clinical
Chemistry (AACC). In their monthly specialist journal Clinical Chemistry
they publish a clinical case study and you can find the archive of these
along with a student discussion document and commentaries from experts
in the field all conveniently located at this website. Do bear in mind that
this is a US site and you may find that some of the terminology and the
units used will be unfamiliar. Nevertheless, it is an excellent resource for
those looking for more advanced examples.

www.labtestsonline.org.uk

This site is described as: “a public resource on clinical lab testing from the
laboratory professionals who do the testing.” The UK site is produced by
the Association of Clinical Biochemists (ACB) – one of the main
professional bodies in the UK, while the US site is produced by the
AACC. Although the site is primarily geared towards a lay audience it
does contain a very extensive listing of laboratory tests and a useful
summary of each including why and how it is performed.

www.acb.org.uk/docs/Article%20Summary%20Use%20of%20HbA
1c%20WHO%20guidance%20251111[1].pdf

This link will take you to a very useful article on the diagnosis of diabetes,
which, as we pointed out in the answer to Case 30, is currently under
review globally.

116
USEFUL WEBSITES

www.aacb.asn.au/web/Education/NITTYs/

If you are looking for more advanced topics a very useful site is that
provided by the Australasian Association of Clinical Biochemists where
you will find an archive of NITTYs (the acronym for Not In The Textbooks
Yet. The site has links to a wide range of presentations that are slide-video
talks of about 10 minutes duration with topics selected for short, discrete
and currently relevant material.

www.metbio.net/metbioTraining.asp

This is the site of the National Metabolic Biochemistry Network and


contains a lot of useful information including a set of case reports –
mostly of inborn errors of metabolism.

117
118

You might also like